You are on page 1of 84

ANALYTICAL GEOMETRY

"Geometry is the right foundation of all painting"

– Albercht Durer
218 | Pioneer Mathematics Grade XI (Approved by CDC Nepal)
3.1

Learning Objective

After the completion of this chapter, students will be able to understand


 find the length of perpendicular from a given point to a given line.
 find the equation of bisectors of the angles between two straight lines.

(Approved by CDC Nepal) Analytical Geometry | 219


The word 'geometry' was derived from the Greek word 'ge' and 'metria' whose meanings are
'earth' and 'measure' respectively. The Cartesian plane is one of the greatest inventions in
mathematics. Greek mathematician Euclid (300–260 B.C.E.) formalized the study of geometry.
Rene Descartes had invented the rectangular coordinate system. The coordinate geometry
enabled mathematicians to 'marry' geometry and algebra.
After the invention of the rectangular coordinate system, algebraic representations became
available to points, lines, curves and other geometric constructions and drawing.
One of the application of the coordinate system and perhaps the most practical is the
mapping of the earth into latitude and longitudes. The mapping was believed to be first
proposed by Amerigo Vespucci, the man who discovered the New World.
Another practical application of coordinate systems is the invention of pixel in computers and
television screens. Each pixel in the screen has its own coordinate, where in most system, the
corner at the upper-left is (0, 0) and both axes are positive. Hence, the coordinates (100, 20)
means that you travel 100 pixels to the right and then 20 pixels down.

Review of Straight line

Distance Formula
Let P(x1,y1) and Q(x2,y2) are any two points on the co–ordinate plane. Let us join PQ.
Then, distance between the points P(x1,y1) and Q(x2,y2) is given by
D = PQ = (x2 – x1)2 + (y2 – y1)2
Remark:
Distance between a point P(x1,y1) from the origin O(0,0) is given by OP = x12 + y12 .
Section Formula
The formula that gives the co–ordinates of a point which divides the join of two points in the
given ratio is called section formula. Section formula gives two types of division:
(i) Internal Division
The, co–ordinate of a point that divides the join of (x 1,y1) & (x2,y2) internally in the ratio
of m1:m2 is given by
mx +mx my +my
(x, y) =  1m2 + m2 1  1m2 + m2 1
 1 2 1 2 

(ii) External Division


The co–ordinate of a point that divides the line joining P(x 1,y1) and Q(x2,y2) in the ratio of
m1:m2 externally is given by,
mx –mx my –my
(x, y) =  1m2 – m2 1  1m2 – m2 1
 1 2 1 2 

Remarks:
(1) The co–ordinates of the midpoint of the line joining the points (x 1,y1) and (x2,y2) is
x +x y +y
(x, y) =  1 2 2  1 2 2 .
 
(2) The co–ordinates of a point that divides the line joining the points (x 1,y1) and (x2,y2) in
kx + x ky + y
the ratio of k:1 is given by (x, y) =  k2+ 1 1  k2+ 1 1 .
 
220 | Pioneer Mathematics Grade XI (Approved by CDC Nepal)
(a) If k >0, then the division is internal.
(b) If k<0 , then the division is external.
(c) If k = 1. then the point is the midpoint.
Equation of straight line in different form
(a) Equation of x-axis
The ordinate of any point on x-axis is zero. i.e y = 0. So the equation of x-axis is y = 0.
(b) Equation of y-axis
The abscissa of any point on the y axis is zero i.e. x=0. So, the equation of y axis is x= 0.
(c) Line parallel to x-axis
Equation of a straight line parallel to x–aixs at a distance 'b' units from it is y = b.
(d) Line Parallel to y axis:
Equation of straight line parallel to y axis at a distance a units from it is given by x=a.
Equation of Straight line in Standard Forms
a. Slope Intercept form
The equation of straight line having slope m and making y intercept c is y = mx+c.
Remarks:
(i) If the line passes through the origin, then c= 0. So the equation of straight line passing
though origin is y = mx.
(ii) If the line is parallel to x axis, then  = 0 , So, slope (m) = 0. Then the equation of the
line will be y = c.
(iii) The equation of a line can't be written in slope intercept form if it is parallel to y axis.
(b) Double intercepts form
x y
The equation of line in double intercepts forms is a + b = 1

Remark:
Any line which doesn't cut x and y axes (either horizontal or vertical) can't be represented by
the double intercept form.
(c) Perpendicular form Y

The required equation of straight line in perpendicular


B
form is xcos+ysin= p, where  is the angle made by
a line perpendicular to the line through the origin and p M
p
is the length of perpendicular from origin to that line.  A
X' X
O
A

Y'

Remarks:
(i) As any line parallel to y axis at a perpendicular distance p from the origin has the
equation x = p as the perpendicular OM makes angle 0° with positive x axis.
(ii) As any line parallel to x axis at a perpendicular distance p from the origin has the
equation y = p as the perpendicular OM makes angle (= 90°) with positive x axis.

(Approved by CDC Nepal) Analytical Geometry | 221


(d) Point slope form
The equation of a line whose slope is m and passing through the point (x1, y1) is y–y1 =
m(x–x1)
(e) Two points form
The equation of a line which passes through the points (x 1, y1) and (x2,y2) is
y –y
y – y1 = x2 – x1 (x –x1)
2 1

First Degree Linear Equation


An equation of the form Ax + By + C = 0, where A, B and C are constant and A, B not
simultaneously zero is called first degree linear equation in x and y.
The first degree linear equation Ax + By + C = 0 always represent a straight line
Reduction of Linear Equation to Three Standard Forms
We reduce the general equation of line Ax + By + C = 0, into three standard form:
(i) Reduction of linear equation Ax+By+C=0 into slope intercept form y=mx+c.
The reduction of line Ax+By+C = 0 in to slope intercept form y = mx + c is
A C
y = – B  x +  – B
   
–A –coeff. of x
Where slope (m) = B = coeff. of y

–C constant
and y– intercept (c) = B = coeff. of y

(ii) Reduction of linear equation Ax + By + C = 0 into double intercepts form:


x y x y
The reduction of line Ax+By+C=0 in double intercept form a + b = is –C/A + –C/B = 1

C
where, x intercept (a) = – A

C
and y intercept (b) = – B

(iii) Reduction of the linear equation Ax + By + C = 0 into the Normal form.


The reduction of Ax+By + C = 0 in to narmal form x cos +y sin  + y sin = p is 
A B C
x+ y=– if C < 0.
A + B2
2
A + B2
2
A + B2
2

–A B C
and x– y= if C < 0.
A2 + B 2 A + B2
2
A + B2
2

A B C
Where, cos = , sin = and p = –
A2 + B 2 A2 + B 2 A2 + B 2
Point of intersection of two straight lines
Let, the equation of two straight lines be

222 | Pioneer Mathematics Grade XI (Approved by CDC Nepal)


A1x + B1y + C1 = 0..............(i)
A2x + B2y + C2 = 0............(ii)
The co–ordinate of the point of intersection of two given lines (i) and (ii) is,

(x, y) = B1 C2 – B2 C1 A2 C1 – C2A1  Provided that A1B2 – A2B1 0


 A1B2 – A2B1 A1B2 – A2B1 
Concurrent Lines
C
Three or more than three lines are said to be concurrent lines if E

they pass through a single common point. Hence the


concurrent lines intersect at a common point which is called
the point of concurrency. In figure, three straight lines AB, CD A B
O
and EF are called concurrent lines as they meet at a common
point O. Point O is called the point of concurrency.
F
Condition of concurrency of three straight lines D

Let the equations of the straight lines be


A1x + B1y + C1 = 0 … … … (i)
A2x + B2y + C2 = 0 … … … (ii)
A3x + B3y + C3 = 0 … … … (iii)
Then the condition of can currency of three straight line is,
A3(B1C2 – B2C1) + B3(A2C1 – B1C2) + C3(A1B2 – A2B1) = 0
and in determinant form.
 A1 B1 C1 
i.e.  A2 B2 C2  = 0
 A3 B3 C3  A
Some Important Points of Concurrency
Centroid of a Triangle P Q
G
The Point of intersection of the three medians of a triangle is called
centroid of the triangle.
Let A(x1,y1), B(x2,y2) and C(x3,y3) be three vertices of the triangle B C
R
ABC. Then the co–ordinate of the centroid G is given by
x1 + x2 + x3 y1 + y2 + y3
(x, y) =  
 3 3 
In–centre of a Triangle A
The point of intersection of the bisectors of the internal angles of a
triangle is called In–centre of the triangle. If AP, BQ and CR are the
Q
bisectors of BAC, ABC and BCA respectively, then the point of R
I
intersection of AP, BQ and CR is the in–centre of ABC. So in
figure I is the In–centre of ABC.
B C
P

(Approved by CDC Nepal) Analytical Geometry | 223


Orthocenter of a Triangle
The perpendiculars from the vertices of a triangle to their opposite A
sides meet at a common point, the common is point called
orthocenter of the triangle.
Let, AP BC, BQ  AC, CR  AB R Q
O
Then point of intersection of AP, BQ and CR is called orthocenter of
ABC. So in figure O is the orthocenter of  ABC.
B C
P

Circumcentre of a Triangle A
The point of intersection of the perpendicular bisectors of the three
sides of a triangle is called circumcentre of the triangle.
Let OP, OQ and OR are the perpendicular bisectors of the sides BC,
O
AC and AB respectively. Then the point of intersection of OP, OQ
and OR is called circumcentre of ABC so in figure. O is the
circumcentre of ABC. B C

Remarks:
i) In case of an equilateral triangle, orthocenter, circumcentre, centroid and incentre
represent the same point.
ii) In other case, centroid of a triangle divides line joining orthocenter and circumcentre of a
triangle in the ratio 2:1.
iii) Centroid of triangle divides the medium of triangle in the ratio of 2:1.
Equation of a straight line through the intersection of two given lines
Let, two lines be
A1x + B1y + C1 = 0.............(i)
A2x + B2y + C2 = 0............(ii)
Then the equation of a straight line through the intersection of (i) and (ii) is given by
A1x + B1y + C1 + K(A2x + B2y + C2) = 0
where K is a constant to be determined under the given conditions.
If  is the angle between two lines y = m1x+c1 and y = m2x+c2 then angle between two lines
m1–m2 
is,  = tan–1  1+m 
 1m2

Remarks:
(i) Positive sign(+) is taken to find the acute angle between the two lines.
(ii) Negative sign(–) is taken to find the obtuse angle between the two lines.
(iii) Two lines will be perpendicular if m1m2 = –1 i.e. if the product of their slope is –1.
(iv) Two lines will be parallel if m1=m2 i.e. if their slopes are equal.
The two lines will be perpendicular to each other if m1.m2 = –1

224 | Pioneer Mathematics Grade XI (Approved by CDC Nepal)


Condition of Parallelism
The two lines will be parallel if m1 = m2
Remarks:
(i) If, A1x + B1y + C1 = 0............(i)
A2x + B2y + C2 = 0..................(ii)
be the equation of straight lines in general forms.
Then, the angle between the two lines is given by
AB –AB
 = tan–1  A 1A 2 + B2 B1 
 1 2 1 2

(ii) The two lines will be perpendicular. if A1A2 + B1B2 = 0


A B
(iii) The two lines will be parallel to each. i.e. if A1 = B1
2 2

Perpendicular Distance of a Straight Line from the Given Point


(i) When the line is in the normal form, xcos + ysin = p
Let xcos + ysin = p be the equation of a straight line AB, the length of perpendicular from
the origin on AB is ON = p and ON makes an angle  with the x axis at origin. i.e AON = 
Let, P(x',y') be any point, draw
PQAB, then PQ is the length of perpendicular D
from Pon line AB. Parallel to line AB and
passing draw a straight line CD through the
point P. So that the length of perpendicular B
from O to the line CD is OM = p1. P (x', y')

As line CD is parallel to line AB the equation N


of the straight line CD is given by Q
P
xcos + ysin = p' .......(i)

Since CD line i.e. line (i) passes through P(x',y'). O A C

x'cos + y'sin = p'.......(ii) Y

In fig(i) OM > ON, so D

PQ = MN = OM – ON = p'– p
In fig(ii) OM < ON, so
B
PQ=ON–OM = p – p' = –(p'–p) M
P (x', y')

Hence, PQ =  [p'– p] = |p' – p| N

 PQ = |x'cos + y'sin – p| which is the P


Q

required perpendicular distance of the  X


straight line xcos + ysin = p from the O A C

given point p(x1–y1).


(ii) When the line is in the form Ax + By + C = 0
Let Ax + By + C = 0...........(i) be the equation of the straight line AB.

(Approved by CDC Nepal) Analytical Geometry | 225


Y

(0, -C/B)
B P (x', y)

X' X
A (-C/A, 0)

Y'

Reducing equation (i) in the perpendicular form,


– A   B  C
2 x + – 2 y = if C > 0
 A +B2
 A +B 2
A + B2
2

A B C
and  
2 x +
 
2 y = – if C <0............ (ii)
 A +B
2
 A +B2
A + B2
2

Comparing (ii) with the equation of a straight line in normal form xcos+ysin=p is (ii)
A B – C
we get, cos =  2 2 , sin= 3 3 and p = +
A +B A +B A2+B2
1 1
we have, the length of perpendicular fram p(x ,y ) to the line (ii) is
P=|x1cos+y1 find – p|
A B C
=   y' – ∓ 2 2
 A2+B2 A2+B2  A +B 
Ax' + By' + C
=
 A2 + B 2 
Ax'+By'+C
=
 A2+B2 
Alternative method:
Let Ax + By + C = 0.........(i) be the equation of line in general form,
which can be written in the form
Ax + By = – C
A B
or, –C x + –C y = 1
x y
or, C + C = 1............(ii)
–A –B

C C
Let P(x',y') be any point draw PQ AB and join PA and PB. since – A and – B are the x
–C C
intercept and y intercept of AB, the coordinates of A and B are  A  0 and 0 –B
   
respectively.
Now, Area of  ABP

226 | Pioneer Mathematics Grade XI (Approved by CDC Nepal)


1 C C C C
= 2 –A –B – 0.0 + 0.y' – x'.–B + x'.0 – –A.y'
       
2
1 C Cx' Cy' 1 C
= 2 AB + B + A  = .2 AB (Ax' + By' + C)
   
1 C
= 2 |AB | |Ax' + By' + C|.....(iii)

Again, Area of ABP


1
= 2 × Base× Height

1
= 2 × AB × PQ

1
= 2 (0 +C/A)2 + (–C/B –0)2 × PQ

1 C2 C2
=2 A2 + B2 . PQ
1 C2(A2 + B2)
=2 A2B2 .PQ

1 C
= 2 AB A2 + B2 .PQ............(iv)
 
Now, from equations (iii) and (iv), we get
1 C 2 2 1 C
2 AB A + B . PQ = 2 AB |Ax' + By' + C|
Ax' + By' + C
 PQ =  which is the required perpendicular distance drawn from a point
 A2 + B 2 
P(x', y') to the straight line Ax + By + C = 0.
Example 1:
Find the length of perpendicular drawn from (2,3) to the line 3x + 4y + 7 = 0
Solution:
Here, given equation of straight line is
3x + 4y + 5 = 0..........(i) and given point (x', y') = (2,3)
Now, Length of perpendicular from (2,3) to the line (i) is
Ax' + By' + C 3.2 + 4.3 + 5 23
P= = = 5
 A2 + B 2   32 + 42 
Example 2:
Find the distance between the parallel lines 2x – 5y –6 = 0 and 6x –15y + 11 = 0
Solution:
Here, given equation of straight lines
2x – 5y –6 = 0...........(i)
6x – 15y + 11 = 0..........(ii)
When x = –2 in equation (i) then , y = –2

(Approved by CDC Nepal) Analytical Geometry | 227


Now,  distance of line (ii) from the point (–2,–2) is,
Ax' + By' + C 6.(–2) –15(–2) + 11 –12 + 30 + 11  29
P= = = =
 A 2
+ B 2
  6 2
+ (–15)2
  36 + 225  261
Example 3:
x y
If p is the length of perpendicular drawn from the origin to the line a + b = 1 ,

1 1 1
prove that a2 + b2 = p2 .

Solution:
x y
Here, given equation of straight line is a + b = 1

1 1
or, a x + b y – 1 = 0 ............(i)
Now, length of perpendicular from origin (0,0) to the line (i) is
1 1
Ax' + By' + C a .0 + b.0 –1  1
p=  =
  =
 A +B 
2 2
1 1 1 1
 a2 + b2  a2 + b2
1
Squaring both sides, we get p2 = 1 1
a2 + b2
Taking reciprocal we get,
1 1 1
∴ a2 + b2 = p2 is the required condition.
Example 4:
Prove that the distance between two parallel lines Ax+By+C 1=0 and Ax+By+C2=0 is
|C2 – C1| Y
d=
A2 + B2
Solution: L2
Here, the given lines are; L1

L1 : Ax+By+C1=0 … … … (i) B
X' X
and L2 : Ax+By+C2=0 … … … (ii) A
–C –C1 ,0
Putting y = 0 in (i), we get x =  A 1 A
 
Y'
 C1 
Thus, the line L1 intersects the x-axis at Q   ,0  .
 A 
 C1 
Then the length of perpendicular from Q   ,0  on line the L2 is,
 A 

228 | Pioneer Mathematics Grade XI (Approved by CDC Nepal)


 C1 
A   0  C2
 A  |C2  C1 |
P= 2 2

A B A2  B2

Equations of the Bisectors of the Angles between Two lines


Let A1x + B1y + C1 = 0 and A2x + B2y + C2 = 0 be the equations of two straight lines AB and
CD respectively. Let P(x', y') be any point on any one of the bisectors (say B1). From P draw
PMAB and PN  CD. Then
PM = PN...........(i) [∵ Two lines are at a equi distance from P]
Y

D
B
M

B1
P(x', y')
S

N
C
A

X' X
O

Y'

A1x' + B1y' + C1
Where, PM = Perpendicular distance of AB from P(x', y') = 
A12 + B12
A2x' + B2y' + C2
and PN = Perpendicular distance of CD from P(x',y') = 
A22 + B22
Hence, from equation (i) we have,
A1x' + B1y + C1 A x' + B2y' + C2
 = 2
A12 + B12 A22 + B22
A1x' + B1y' + C1 A x' + B2y' + C2
or, 2 2 = 2 ................. (ii)
A1 + B1 A22 + B22
Equations (ii) are true for any point (x',y') on any of the bisectors. Hence
A1x + B1y + C1 A x + B2y + C2
∴ =  2 are the required equations of the bisectors of the
A12 + B12 A22 + B22
angle between two lines.
Remarks: To find the bisector of the acute angle
Let  be the angle between any of the bisectors say B1 and any one of the line, say AB then
(i) If |tan| < 1 then B is bisector of the acute angle and B2 is bisector of obtuse angle.
(ii) If |tan| > 1, then B, is bisector of obtuse angle and B2 is bisector of acute angle.

(Approved by CDC Nepal) Analytical Geometry | 229


Equation of the bisector of the angle containing the origin.
Let A1x + B1y + C1 = 0 and A2x + B2y + C2 = 0 where (C1C2>0 be the equations of the lines
AB and CD. Hence if origin (0,0) is substituted in above equations both of them will be
positive. Then the co–ordinates of any point lying on the bisector containing origin makes A1x
+ B1y + C1 Positive as the point and the origin lies on the same side of AB. similarly A2x +
B2y + C2.
Therefore, equation of the bisector of an angle containing the origin is
A1x + B1y + C1 A2x + B2y + C2
=
A12 + B12 A22 + B22
Note that the equations of the bisector of the angle not containing the origin is
A1x + B1y + C1 A x + B2y + C2
=– 2
A12 + B12 A22 + B22
Example 5:
Find the equation of the bisectors of the angles between the lines.
x – y + 4 = 0 and 7x – y + 8 = 0. Also, determine the bisector of the acute angle.
Solution:
Here, given equations of the lines are
x – y + 4 = 0..............(i)
7x – y + 8 = 0..........(ii)
Now, equations of the bisectors the angles between the lines (i) and (ii) are
A1x + B1y + C1 A x + B2y + C2
= 2
A12 + B12 A22 + B22
x–y+4 7x – y + 8
 =
(1)2 + (–1)2 72 + (–1)2
x–y+4 7x – y + 8
or, = )
2 49 + 1
x–y+4 7x – y + 8
or, =
2 50
x–y+4 7x – y + 8
or, =
2 2 × 25
7x – y + 8
or, x – y + 4 =  5
Taking + ve sign Taking – ve sign
7x – y + 8 7x – y + 8
x–y+4= 5 x–y+4=– 5
or, 7x – y + 8 = 5x – 5y + 20 or, 5x – 5y + 20 = –7x + y – 8
or, 2x – 4y – 12 = 0 or, 12x – 6y + 28 = 0
or, x + 2y – 6 = 0 or, 6x – 3y + 14 = 0
Hence, required equations of the bisectors are
x + 2y –6 = 0 .....(iii)

230 | Pioneer Mathematics Grade XI (Approved by CDC Nepal)


and 6x – 3y + 14 = 0...........(iv)
To find the bisectors of the acute angle.
1
Slope of line (i) is m1 = – –1 = 1

–1 –1
and slope of bisector (iii) is m2 = 2 = 2

Now, If  be the angle between the line (i) and bisector (iii) , then
1
m – m 1+2 
|tan| = 1 +1 m .m 2 
=   = 3.
 2 –1
1 + 1. 2 
 
1
 
Since |tan| = 3 > 1, bisector (iii) is the obtuse angle bisector and hence bisector (iv) is
the acute angle bisector.
Hence equation of the bisector of acute angle is 6x –3y + 14 = 0.
Example 6:
Find the equation of the bisector of the angle between the lines which contains the
origin.
x –7y + 5 = 0 and x + y –3 = 0
Solution:
Here, given equations of the lines
x – 7y + 5 = 0 ............(i)
and x + y – 7 = 0.............(ii)
The equation of the bisector of the angle between two lines containing the origin is
A1x + B1y + C1 A2x + B2y + C2
=
A12 + B12 A22 + B22
x – 7y + 5 x+y–3
or, 2 2 =
(1) + (–7) 12 + 12
x – 7y + 5 x + y –3
or, =
50 2
x – 7y + 5
or, 5 = x + y –3

or, 5x + 5y –15 = x – 7y + 5
or, 4x + 12y –10 = 0 is the required equation of bisector that contains the origin.
Example 7:
Find the value of P if the length of the perpendicular from the point (1,–1) on the line
3x –2y + P = 0 is 5 units.
Solution:
Here, given equation of the straight line is
3x –2y + P = 0......(i)
Now, perpendicular distance from the point (1,–1) on the line (i) is
3.1 –2.(–1) + P
P = 
 32 + (–2)2 
(Approved by CDC Nepal) Analytical Geometry | 231
5+P
or, ±5 =
13
or, 5 + P = ±5 13
 P = 5(± 13 –1)
Example 8:
The length of perpendicular drawn from the point (r,3) on the line 3x + 4y + 5 = 0 is 4.
Find the value of r.
Solution:
Here, given equation of the line is
3x + 4y + 5 = 0............(i)
Now, perpendicular distance of the line from the point (r,3) is
3r + 4.3 + 5
P=
32 + 42
3r + 17
or, 4 =  5
or, 3r + 17 = 20
or, 3r =  20 –17
Taking + ve sign Taking – ve sign
3r = 20–17 3r = –20 –17
or, 3r = 3 or, 3r = –37
37
or, r = 1 or, r = – 3
37
∴ The values of r are 1 and – 3 .
Example 9:
If p and p' be the length of the perpendiculars from the origin upon the straight line
whose equations are xsec + ycosec = a and xcos – ysin = acos2 Prove
that 4p2 + p'2 = a2.
Solution:
Here, given equation of straight lines are
xsec + ycosec – a = 0..........(i)
xcos – ysin – acos2 = 0.......(ii)
Since p and p' are the length of the perpendiculars from the origin (0,0) on the lines (i)
and (ii) respectively, we have,
0.sec + 0.cosec – a
p=
sec2 + cosec2
a
or, p =
1 1
+
cos2 sin2
a
or, p =
sin  + cos2
2

sin2.cos2
or, p = asin.cos.............(iii)

232 | Pioneer Mathematics Grade XI (Approved by CDC Nepal)


0.cos – 0.sin – acos2
and, p' = 
cos2 + (–sin)2
or, p' = – acos2............(iv)
Now, 4p2 + p'2 = 4(asin.cos)2 + (–acos2)2 = 4a2 sin2.cos2 + a2cos22
= a2(4sin2.cos2 + cos22) = a2(sin22 + cos22)
 4p2 + p'2 = a2 Proved.
Example 10:
Find the points on the y–axis each of whose perpendicular distance from the line
ax + by + c = 0 is b.
Solution:
Here, given equation of the line is
ax + by + c = 0...........(i)
So, perpendicular distance of the line (i) from the point on y–axis (0,y) is
a.0 +b.y + c
P=
a2 + b2
by + c
or, b = 
a2 + b2
or, by + c =  b a2 + b2
or, by = – c  b a2 + b2
1
or, y = b (–c  b a2 + b2 )

1
Hence, required points on the y –axis are 0 b{–c b a2 + b2
 

EXERCISE 3.1
1. (a) Determine whether the points (2,3) and (3,–2) are on the same side of the line
2x – y + 1 = 0
(b) Are the points (2,3) and (1,3) on the same side or on opposite sides of the line
x – 2y + 3 = 0?
2. Find the length of perpendiculars draw from
(a) (1,1) to the line 4x + 3y –12 = 0
(b) (2,1) to the line x –2y –5 = 0
x y
(c) (b,a) to the line a – b = 1

(d) (4,3) to the line perpendicular to the line 4x –3y –10 = 0 and passing through (5,4).
3. Find the distance between the parallel lines
(a) 3x + 4y = 5 and 6x + 8y –45 = 0
(b) 3x + 5y –11 = 0 and 3x + 5y + 23 = 0
(c) 12y = 5x + 4 and 5x = 12y –3

(Approved by CDC Nepal) Analytical Geometry | 233


x y
4. If p is the length of perpendicular dropped from the point (a,b) on the line a + b = 1 ,
1 1 1
prove that a2 + b2 = p2

5. (a) The perpendicular distance from the point (3,–5) on the line kx + 4y + 7 = 0 is 3
units, find the value of k.
(b) The perpendicular distance of the line 3x + 4y + 5 = 0 from the point (a,3) is 4, find
the value of a.
6. Find the equation of the bisectors of the angles between the following lines
(a) x –2y = 5 and 11x –2y + 6 = 0
(b) 3x – y + 1 = 0 and 3x – 9y = 7
(c) 3x – 2y + 1 = 0 and 18x + y –5 = 0,
Also identify the bisector of acute angle and the bisectors of the angle between the lines
containing the origin.
7. Find the equation of the bisectors of the angles between the lines 3x + 4y + 2 = 0 and
12x –5y + 3 = 0. Show that the bisectors are at right angles.
8. Find the equation of the bisector of the acute angle between the lines 2x + y = 4 and 4y
+ 2x = 5.
9. Find the co–ordinates of the points on the x–axis whose perpendicular distance from the
x y
straight line a + b = 1 is c.

10. The equations of two sides of a square are y + 2x = 0 and y + 2x = 3 and the origin is a
corner of it. Find the equations of other two sides.
11. If p and p' are the length of the perpendiculars drawn from the points (cos, sin) and (–
4
sec, cose) on the line xsin + ycos = 0 respectively, prove that p2 – p'2 = 4

12. Show that the product of the perpendiculars drawn from the points ( a2 – b2 ,0) on the
x y
line a cos + b sin = 1 is b2.

13. (a) The line x –5y = 7 bisects an angle between a pair of lines one of which is
8x – y = 3. Show that the other line is 7x + 4y + 4 = 0.
(b) The line 11x –3y = 0 bisects an angle between a pair of lines one of which is
3x – 2y + 1 = 0. Show that the other line is 18x + y = 5.
14. Find the equations of straight lines each of which is parallel to and at a distance of 5
from the line x + 2y –7 = 0.
15. Find the equations of the two straight lines drawn from through a point (0,a) on which
the perpendiculars drawn from the point (2a,2a) are each of length a.
16. Find the equations of the straight line which is at right angles to 3x + 4y –12 = 0 such
that its perpendicular distance from the origin is equal to the length of the perpendicular
from (3,2) on the given lines.
17. The points (5,1) and (1,3) are the opposite vertices of a rectangle. The other two vertices
lie on the line y = 2x + c. Find c.

234 | Pioneer Mathematics Grade XI (Approved by CDC Nepal)


ANSWERS

1. (a) same side (b) same side


b2 – a2 – ab 7
2. (a) 1 (b) 5 (c) (d)  5
b2 + a2
7 1
3. (a) 2 (b) 34 (c) 13

25 37
5. (a) 78 (b) 1 or – 3

6. (a) 6x + 8y + 31 = 0, 16x –12y – 19 = 0 (Acute angle bisector)


(b) 3x + 3y + 5 = 0, 3x –3y –1 = 0 (Acute angle bisector)
(c) 11x –3y = 0, 3x + 11y – 10y = 0 (Acute angle bisector)
7. 99x + 27y + 41 = 0, 21x –77y –11 = 0

8. 6x + 6y = 13 9. a  c a2 + b2  0
 b 
10. x –2y = 0, x – 2y  = 5 14. x + 2y –2 = 0, x + 2y –12 = 0
15. y =a, 4x – 3y + 3a = 0 16. 4x –3y = 5
17. –4

Objectives Questions
Circle the correct Answers
1. The ratio in which the line 3x-y+5=0 divides the line segment joining the points (2,5) and (-2,2) is
a. 1:2 b. 2:1 c. 3:2 d. 1:3
2. Mid points of the sides AB and AC of ABC are (3,5) and (-3,-3) respectively then the length of the
side BC is
a. 5 b. 10 c. 20 d. 25
3. A straight line meets the axes of A and B such that the centroid of ABC is (a,a). Then the equaiton
of the line AB is
a. x+y b. x-y = 3a c. x+y = 2a d. x+y=3a
4. The distance between points p(-3,-2) and Q(-6,7), the axis being inclined at 60° is.
a. 7 b. 3 7 c. 2 7 d. None
5. The points (k,2-2k), (-k+1,2k) and (-4-k,6-2k) are collineer for
1
a. all values of K b. No value of k c. k = -1 d. k = -1 or
2
6. If (3,3) lies on the line joining the points (h,o) and (o,k) then
1 1 1
a. h+k = 9 b. + = c. hk = 3 d. 3h-3k = 1
h k 3
7. Two points (a,0) and (0,b) are joined by a straight line another point in this line is
a(3a, -2b) b. (a,-b) c. (-2a,b) d. (2b,-a)

(Approved by CDC Nepal) Analytical Geometry | 235


8. If the point (x,y) be equidistant form the points (a+b, b-a) and (a-b, a+b) then
a. ax_by = 0 b. ax-by = 0 c. bx+ay = 0 d. bx-ay = 0
9. If the lines x + 2ay+a = 0, x + 3by + b = 0 and x+ 4cy+c = 0 are cuncurrent then a,b,c are in
a. A.P b. G.P c. H.P d. None
10. The opposite vertices of a square are (a,a) and (-a,-a) then other vertices are
a. (-a,a),(a,-a) b. (a,-a),(-a,o) c. (a,0),(-a,0) d. (0,a),(0,-a)
11. Which of the two 3x - 4y + 4 = 0 and 4x - 3y + 12 =0 is nearer to origin
a. 4x-3y-12=0 b. 3x-4y+4=0 c. both a and b c. None
12. Equaitonof a line parallel to 2x+3y+11 = 0 such that sum of its intercept on the axes is 15 is
a. 2x-3y+18=0 b. 2x+3y+18=0 c. 2x-3y-18=0 d. 2x+3y-18=0
13. The circumstance of a rt. angled triangle with vertics A(3,0), B(0,4) and C(0,0) is
3
a. (0,0) b. (3,4) c. (3,-2) d. ( ,2)
2
14. The orthocentre of a triangle is (-3,5) and the circumstance is (0,2). then the centroid is
a. (1,2) b. (3,3) c. (-2,3) d. 1.-4)
15. (0,0) is the vertex of a square and the equaltion of one of its sides is 5x-12y+26 = 0. Then the area of
the square is
a. 4 sq. units b. 8 sq. units c. 12 sq. units d. 16 sq. units
16. The equation of the straight line equality inclined to the axes equidistant from the point (1,-2) and
(3,4) is
a. x+y+1 = 0 b. x+y+2 = 0 c. x-y-2=0 d. x-y-1 = 0
17. The equation y-y1 = m (x-x1), mR represents all lines through (x1,x) except the line
a. Parallel to x-axis b. Parallel to y-axis
c. parallel to line x-y=0 d. Parallel to line x+y=0
18. Projection (the foot of perpendicular) from (x,y) on the x-axis is
a. (x,0) b. (0,y) c, (-x,0) d. (0,-y)
19. The line x+y-6 = 0 is the right bisactor of the segment PQ. If p is the point (4,3) then the point Q is
a. (4,4) b. (3,3) c. (3,4) d. (3,2)
20. Two opposite vertices of a rectangle are (1,3) and (5,1). if the equaiton of a diagonal of this
rectangle is y = 2x+c then the value of c
a. -4 b. 1 c. -9 d. -5

Answers:
1 b 2 c 3 d 4 b 5 d
6 b 7 a 8 b 9 c 10 a
11 b 12 d 13 d 14 b 15 a
16 d 17 b 18 a 19 d 20 a

236 | Pioneer Mathematics Grade XI (Approved by CDC Nepal)


3.2

Learning Objective

After the completion of this chapter, students will be able to understand


 write the condition of general equation of second degree in x and y to represent a pair of
straight lines.
 find angle between pair of lines and bisectors of the angles between pair of lines given by
homogenous second degree equation in x and y.

(Approved by CDC Nepal) Analytical Geometry | 237


General equation of second degree
Let, A1x + B1y + C1 = 0 ...............(i)
and A2x + B2y + C2 = 0...............(ii)
Be the equations of two straight lines.
Consider the combined equation
(A1x + B1y + C1) (A2x + B2y + C2) = 0..............(iii)
The co–ordinate of any point in (i) or (ii) satisfy the equation (iii) and vice versa.
Hence equation (iii) is a single equation representing a pair of straight lines (i) and (ii).
The equation (ii) can be written as A1A2x2+(A1 + B2A2B1) xy + B1B2y2+ (A1C2 + A2C1) x
+ (B1C2 + B2C1)y + C1C2=0.............. (iv)
Let A1A2 = a, A1B2+A2B1 = 2h B1B2 = b, A1C2+A2C1 = 2g, B1C2+B2C1 = 2f and C1C2 = C.
Then equation (iv) can be written as ax2 + 2hxy + by2 + 2gx + 2fy + c = 0 .........(iv)
Equation is called general equation of second degree in x and y.
Remarks:
The equation of a pair of straight lines can be represented by the general equation of second
degree but the converse is not always true, i.e. general equation of second degree may not
represent a pair of straight lines. For example, x2 + y2 = a2 is an equation of 2nd degree, which
does not represent two straight lines.
Example 1:
Find a single equation representing the line pair x + y + 2 = 0 and x + 2y + 1 = 0.
Solution:
Here, given equations of straight lines are,
x + y + 2 = 0.........(i)
and x + 2y + 1 = 0...........(ii)
Combining equations (i) and (ii), we get
(x + y + 2) (x + 2y + 1) = 0
or, x2 + 2xy + x + xy + 2y2 + y + 2x + 4y + 2 = 0
or, x2 + 2y2 + 3xy + 3x + 5y + 2 = 0
or, x2 + 3xy + 2y2 + 3x + 5y + 2 = 0 which is the required single equation
representing the line pair.
Example 2:
Find the separate equations of the two lines represents by the equation.
x2 + 2xy + y2 –2x –2y –15 = 0
Solution:
Here, given equation of line pair is
x2 + 2xy + y2 –2x –2y –15 = 0
or, (x + y)2 –2(x + y) –15 = 0
or, (x + y)2 – 5(x + y) + 3(x + y) –15 = 0

238 | Pioneer Mathematics Grade XI (Approved by CDC Nepal)


or, (x + y) (x + y –5) + 3(x + y–5) = 0
or, (x + y–5) (x + y + 3) = 0
 x + y –5 = 0 and x + y + 3 = 0 are the two separate equation of straight lines.
Second Degree Homogeneous Equation
An equations in the form, ax2 + 2hxy + by2 = 0 called second degree homogeneous equation
in x and y.
Theorem:
Prove that the homogeneous equation of second degree always represents a pair of straight
lines passing through the origin.
Consider second degree homogeneous equation
ax2 + 2hxy + by2 = 0...........(i)
Case I: When b 0 , then divide the equation (i) by b, we get
2h a
y2 + b xy + bx2 = 0
2

or, y + 2h y + a = 0 .................(ii)


x b x b
y
This is quadratic equation in x which has two values, say m1 and m2.

y y
Then, x = m1 and x = m2

or, y = m1x and y = m2x are the straight lines passing through the origin
Case II: When b = 0, then equation (i) can be written as
ax2 + 2hxy = 0
or, x(ax + 2hy) = 0
 x = 0 and ax + 2hy = 0 are the straight lines passing through the origin.
Hence, homogeneous equation of 2nd degree always represents a pair of straight lines passing
through the origin.
Case III: If a = 0, then (i) reduces in the form
2hxy + by2 = 0
or, y(2hx + by) = 0
⇒ y = 0 and 2hx+by = 0 are the straight lines passing through the origin.
Remarks:
If the homogeneous equation of second degree
ax2+2hxy+by2=0 i) represent a pair of straight lines. y=m1x......(i) and y=m2x
–2h a
Then, m1 + m2 = b and m1.m2 = b

Example 3:
Find the separate equations of the lines represented by x 2 + 2xysec + y2 = 0.
Solution:
Here, the given equation is
x2 + 2xysec + y2 = 0

(Approved by CDC Nepal) Analytical Geometry | 239


or, y2 + 2xy sec + x2 = 0
2

or, y + 2. y sec + 1 = 0


x x
y
This is quadratic equation in x which has two values m1 and m2 so that

m1 + m2 = –2sec...............(i) and m1.m2 = 1


Now, (m1 –m2) = (m1 + m2) – 4m1.m2 = 4sec2 – 4
2 2
= 4(sec2 –1) = 4 tan2
 m1 –m2 =  2tan.............(ii)
Solving equations (i) and (ii)
2m1 = – 2sec  2tan
or, m1 = – sec  tan
Substituting value of m1 in equation (i)
m2 = –2sec + sec tan
or, m2 = – sec  tan
m2 = sec  tan
Therefore, equation of straight lines are
y = (–sec  tan)x
Taking + ve and – ve sign we get the two separate equations of straight line.
Angle between the line pair represented by ax2 + 2hxy + by2 = 0
If  be the angle between the two lines represented by the second degree homogeneous
2 h2 – ab
equation ax2 + 2hxy + by2 = 0, then tan =  a + b . Also, find the condition that hte
two lines to be perpendicular and parallel.
Proof:
Y

A
y = m1x

B
y = m2x

X' X
O

Y'

Let us consider second degree homogenous equation.


ax2 + 2hxy + by2 = 0............(i)
Which always represent a pair of lines passing through the origin say
y = m1x..............(ii)
and y = m2x..............(iii)

240 | Pioneer Mathematics Grade XI (Approved by CDC Nepal)


2h
Such that m1 + m2 = – b ..............(v)
a
and m1.m2 = b ............(vi)

Let  be the angle between two lines (ii) and (iii), then,
2
–2h – 4. a
m – m2 2
(m1 + m2) – 4m1.m2  b b 2 h2 –ab
tan =  1 +1 m .m = 1 + m1.m2 = a = a+b
1 2
1+b

  = tan–1 
 2 h2 – ab is the angle between the two lines represented by second degree
 a+b 
homogeneous equation.
Condition of Perpendicularity
Two lines will be perpendicular if  = 90°
i.e. if tan = tan90°

i.e. if
 2 h2 – ab = 1
 a+b  0
i.e. if , a + b = 0
Condition of Parallelism
The two lines will be Parallel if  = 0°
i.e. if tan = tan0°
2 h2 – ab
i.e. if a+b =0
i.e. if 2 h2 – ab = 0
i.e. if h2 – ab = 0
i.e. if h2 = ab is required condition of Parallelism.
Remark:
The homogeneous equation of second degree ax2 + 2hxy + by2 = 0 represent a pair of
(a) real and distinct lines if h2 – ab > 0
(b) Coincident lines if h2 – ab = 0
(c) Imaginary lines if h2 – ab < 0.
Example 4:
Find the angle between the line pair represented by the following equations.
(a) 3x2 – 15xy + 2y2 = 0
(b) x2 + 6xysin – y2 = 0
Solution:
(a) Here, given equations of line pair
x2 + 9xy + 14y2 = 0............(i)
Comparing equation (i) with ax2+2hxy+by2=0, we get

(Approved by CDC Nepal) Analytical Geometry | 241


9
a = 1, 2h = 9 or, h = 2 and b = 14

Now, if  be the angle between the line pair represented by equation (i), then
2

2 9 – 1×14
2
2 h – ab 2 2 81 – 56 5 1
tan =  a+b = 1 + 14 = 2×15 =  15 =  3

1
  = tan–13 is the required angle between the line pair.
 
(b) Here, given equation of the line pair
x2 + 6xy sin – y2 = 0...............(i)
Comparing equation (i) ax2+2hxy+by2=0, we get
a = 1, 2h = 6sin or, h = 3sin
 be the angle between the line pair represented by the equation (i), then
2 h2 – ab 2 (3sin)2 – 1× (–1) 2 9sin2 + 1
tan =  a+b =  1–1 =  0 =  = tan90°

  = 90° is the required angle between the line pair.


Equation of the bisectors of the angles between the line pair represented by second
degree homogeneous equation ax2 + 2hxy + by2 = 0.
Theorem:
Prove that, the equations of the bisectors of the angles between the line pair
ax2 + 2hxy + by2 = 0 are given by h(x2 – y2) = (a – b) xy
Proof:
Y

H
Y

F
P (x',y')

y = m2x
X' X

A G

Y'

Consider second degree homogeneous equation


ax2 + 2hxy + by2 = 0............(i)
Let, y = m1x........(ii) and y = m2x..............(iii)
be the equation of the lines represented by the equation (i) then we have

242 | Pioneer Mathematics Grade XI (Approved by CDC Nepal)


2h a
m1 + m2 = – b ...........(iv) and m1.m2 = b ............(v)

Then the equations of the bisectors are given by


m1x – y m2x – y
=
m12 + 1 m22 + 1
Squaring on both sides,
(m1x – y)2 (m2x – y)2
2
m1 + 1 = m22 + 1
m12 x2 – 2m1xy + y2 m22x2 – 2m2xy + y2
or, 2
m1 + 1 = m22 + 1
or, (m22 + 1) (m1x2 – 2m1xy + y2) = (m12 + 1) (m22x2 – 2m2xy + y2)
or, x2(m12 – m22) – 2xy(m1 – m2)(1 – m1.m2) – y2(m12 – m22) = 0
or, (x2 – y2) (m12 – m22) = 2xy(m1 – m2)(1 – m1.m2)
2xy(1 – m1m2)
or, x2 – y2 = m1 + m 2
Now, substituting the values from equations (iv) and (v)
1 – a/b
x2 – y2 = 2xy  2h 
 – 
 b 
(b – a)
or, x2 – y2 = 2xy –2h

(a – b)
or, (x2 – y2) = xy h
or, h(x2 – y2) = (a – b) xy which are the required equation of the bisectors.
Example 5:
Find the equations of the bisectors of the angles between the line– pair.
3x2 – 15xy + 2y2 = 0. Also show that the bisector is at right angle.
Solution:
Here, given homogeneous equation
3x2 – 15xy + 2y2 = 0..........(i)
Comparing equation (i) with ax2 + 2hxy + by2 = 0, we get
15
⇒ a = 3, 2h = – 15 or, h = – 2 and b = 2

Now, equations of the bisectors of the angles between the lines represented by (i) are
h(x2 – y2) = (a – b) xy
–15
or, 2 (x2 – y2) = (3–2)xy

or, –15(x2 – y2) = 2xy


or, –15x2 + 15y2 = 2xy
or, 15x2 + 2xy –15y2 = 0.............(ii)
Again, comparing equation (ii) with ax 2 + 2hxy + by2 = 0
a = 15, h = 1, b = –15

(Approved by CDC Nepal) Analytical Geometry | 243


Now, the condition of perpendicularity is
a+b=0 i.e. 15 –15 = 0
or, 0 = 0 which is true. Hence the bisectors are at right angles.
Example 6:
If the line pairs ax2 + 2hxy + by2 = 0 and a'x2 –2h'xy + b'y2 = 0 have the same bisectors
then prove that h(a' – b') = h'(a – b).
Solution:
Here, given homogeneous equations
ax2 + 2hxy + by2 = 0..............(i)
a'x2 – 2h'xy + b'y2 = 0...........(ii)
Now, equations of the bisectors of the angles between the lines given by (i) are
h(x2 – y2) = (a – b) xy.............(iii)
and equations of the bisectors of the angles between the lines given by (ii) are
h'(x2 – y2) = (a' – b') xy ............(iv)
Since (iii) and (iv) are identical co–efficient of corresponding terms are proportional.
h a–b
i.e. h' = a' – b'

 h(a' – b') = h'(a – b) which is the required condition.


Condition for the General Equation of Second Degree to represent a line pair.
Theorem:
Prove that the general equation of second degree ax2 + 2hxy + by2 + 2gx + 2fy + c = 0
represents a line pair if
abc + 2fgh – af2 – bg2 – ch2 = 0
Proof:
Consider general equation of second degree ax2+2hxy+by2+2gx+2fy+c=0...............(i)
For a 0, equation (i) can be written as
ax2 + 2(hy + g) × by2 + 2fy + c = 0..............(ii)
which is quadratic equation in x, so solving for x we have
–b  b2 – 4ac –2(hy + g)  {–(hy + g) }2 – 4.a(by2 + 2fy + c)
x= 2a x= 2a
1
or, x = a {–(hy + g)  (hy + g)2 – a(by2 + 2fy + c) }

The expression on the right side of above equations is linear


if (hy + g)2 – a(by2 + 2fy + c) be a perfect square
i.e. if (h2 – ab) y2 + 2(gh – af) y + (g2 – ac) be a perfect square.
This is possible when the discriminant b2 – 4ac of expression is zero.
i.e. if 4(gh – af)2 – 4(h2 – ab) (g2 – ac) = 0
i.e. if (gh – af)2 – (h2 – ab) (g2 – ac) = 0
i.e. if a(abc + 2fgh – af2 – bg2 – ch2) = 0
As, a  0, abc + 2fgh – af2 – bg2 – ch2 = 0 ...........(iv) which is the required condition.
If a= 0 and b 0 we can find the condition in the same way.

244 | Pioneer Mathematics Grade XI (Approved by CDC Nepal)


Remark:
(i) The condition abc + 2fgh – af2 – bg2– ch2 = 0 can also be written in the determinant form
a h g
 h b f  =0
 g f c
(ii) ax + 2hxy + by2 + 2gx + 2fy + c = 0 is the product of two linear factors.
2

(iii) If ax2 + 2hxy + by2 + 2gx + 2fy + c = 0 does not represent pair of straight lines, then it
represents the of a curve,
Example 7:
Prove that the general equation 6x2 – xy – 12y2 –8x + 29y –14 = 0 represents a pair of
straight lines.
Solution:
Here, given equation of pair of lines is,
6x2 – xy – 12y2 – 8x + 29y – 14 = 0..........(i)
Comparing equation (i) with ax2 +2hxy + by2 + 2gx + 2fy + c = 0, we get
a = 6, 2h = –1, b = –12, 2g = –8, 2f = 29, c = –14
1 29
⇒ a = 6, h = –2 , b = –12, g = –4 f= 2 c = –14

Using, the condition that the equation (i) represents a pair of straight lines
abc + 2fgh – af2 – bg2 – ch2 = 0
2 2
29 1 29 1
or, 6( –12) (–14) + 2× 2 .(–4)–2 – 6 2  – (–12) (–4)2 – (–14) –2 = 0
     
5046 7
or, 1008 + 58 – 4 + 192 + 2 = 0

5046 7
or, 1258 – 4 +2 =0
or, 5032 – 5046 + 14 = 0
0 = 0 which is true. So, the given equation represents the line pair.
Example 8:
For what value of k the equations 2x2 + xy – y2 + kx + 6y – 9 = 0 represent a pair of
straight lines.
Solution:
Here given equation of pair of lines is,
2x2 + xy – y2 + kx + 6y –9= 0................(i)
Comparing equation (i) with general equation of second degree
ax2 + 2hxy + by2 + 2gx + 2fy + c = 0, we get
1 k
a = 2, h=2, b = –1, g=2, f = 3 and c = –9

Using, the condition for the equation (i) to represent a pair of straight lines
abc + 2fgh – af2 – bg2 – ch2 = 0
2 2
k 1 k 1
or, 2.(–1) .(–9) + 2.3.2 .2 – 2.32 – (–1) 2 – (–9) 2 = 0
   

(Approved by CDC Nepal) Analytical Geometry | 245


3k k2 9
or, 18 + 2 – 18 + 4 + 4 = 0

3k K2 9
or, 2 + 4 +4 =0
or, 6k + k2 + 9 = 0
or, k2 + 6k + 9 = 0
or, (k + 3)2 = 0
For, k = –3, the equation (ii) represent a pair of straight lines.
Relation between General and homogeneous equation of second degree.
Therom:
If the equation ax2 + 2hxy + by2 + 2gx + 2fy + c = 0 represent a pair of straight lines, then
ax2 + 2hxy + by2 = 0 represents a pair of straight lines through the origin and parallel to the
former pair.
Proof:
Let ax2 + 2hxy + by2 + 2gx + 2fy + c = 0...........(i) represent a pair of straight lines.
Then equation (i) can be resolved into two linear factors which can be written as
(l1x + m1y + n1) (l2x + m2y + n2) = 0...........(ii)
or, l1l2x2+(l1m2+l2m1)xy+m1m2y2 + (n1l2 + n2l1)x + (m1n2 + m2n1)y + n1n2 = 0..........(iii)
Equation (i) and (iii) represent same pair of straight lines, so,
l1l2 = a, m1m2 = b, and l1m2 + l2m1 = 2h ...........(iv)
Now, equations of straight lines through the origin and parallel to lines given by (ii) are
(l1x + m1y) (l2x + m2y) = 0
or, l1l2x2 + (l1m2 + l2m1) xy + m1m2y2 = 0
By using equation (iv) we get,
ax2 + 2hxy + by2 = 0
Hence ax2 + 2hxy + by2 = 0 represents a pair of straight lines passing through the origin
and parallel to the lines given by ax2 + 2hxy + by2 + 2gx + 2fy + c = 0.
Remarks:
(i) Since the lines represented by ax2 + 2hxy + by2 + 2gx + 2fy + c = 0.............(i)
are parallel to the lines represented by ax2 + 2hxy + by2 = 0.............(ii)
So, the angles between the line pair (i) and (ii) are same and is given by
2 h2 – ab
tan =  a+b
(ii) The lines represented by (i) are perpendicular if a + b = 0
(iii) Two lines represented by ax2 + 2hxy + by2 + 2gx + 2fy + c = 0 are parallel if h 2 = ab
and af2 = bg2 and coincident if g2 = ac
(iv) The distance between two parallel lines represented by (i) is
g2 – ac f2 – bc
d=2 a (a + b) or, 2 b (a + b)

246 | Pioneer Mathematics Grade XI (Approved by CDC Nepal)


Example 9:
Find the equation of straight lines perpendicular to the lines given by
x2+xy–6y2+7x+31y–18 = 0 and passing through the origin.
Solution:
Here, given equation of pair of straight lines is,
x2 + xy –6y2 + 7x + 31y –18 = 0............(i)
Now, taking the homogeneous part only,
i.e. x2 + xy – 6y2 = x2 + 3xy –2xy –6y2
= x(x + 3y) –2y(x + 3y)
= (x + 3y) (x –2)
Now, the equation of straight lines perpendicular to the lines given by (i) are,
(3x – y + k1) (–2x – y + k2) = 0.............(ii)
Since the lines (ii) pass through the origin. So, k1 = 0 and k2 = 0
Substituting the values of k1 and k2 in equation (ii), we get,
(3x –y) (–2x – y) = 0
or, –6x2 – xy + y2 = 0
or, 6x2 + xy – y2 = 0 which are the required equations.
Alternative method:
Given equation of pair of straight lines is,
x2 + xy – 6y2 + 7x + 31y – 18 = 0.............(i)
Solving for x, we have,
(y + 7)  (y + 7)2 – 4.1(–6y2 + 31y –18)
x=– 2.1
–(y + 7)  y2 + 14y + 49 + 24y2 –124y + 72
or, x = 2
–(y + 7)  25y2 – 110y + 121
or, x = 2
–(y + 7)  (5y – 11)2
or, x = 2
or, 2x = –(y + 7)  (5y – 11)
Taking + ve sign Taking –ve sign.
2x = –y –7 + 5y –11 2x = –y – 7 – 5y + 11
or, 2x – 4y + 18 = 0 or, 2x + 6y –4 = 0
or, x – 2y + 9 = 0............(iii) or, x + 3y –2 = 0...........(iv)
Now, equation of straight line perpendicular to (iii) and (iv) are
–2x –y + k1 = 0 and 3x – y + k2 = 0

(Approved by CDC Nepal) Analytical Geometry | 247


As the lines pass through the origin. So, k1 = 0, and k2 = 0
Required equation of pair of lines is
(–2x – y) (3x – y) = 0
or, –6x2 – xy + y2 = 0
 6x2 + xy – y2 = 0
Equation of straight lines joining the origin to the intersection of a line and a curve
Y

C
Q

X' X
O

Y'

Let a straight line lx + my = n............(i)


and a curve ax2 + 2hxy + by2 + 2gx + 2fy + c = 0...........(ii)
intersect at two points P and Q.
Join OP and OQ. Now we wish to find the equations of OP and OQ.
lx + my
Now, equation (i) can be written as n = 1 ............(iii)

Making equation (ii) homogeneous with the help of equation (iii).


i.e., ax2 + 2hxy + by2 + 2(gx + fy) .1 + c.(1)2 = 0
(lx + my) (lx + my)2
or, ax2 + 2hxy + by2 + 2(gx + fy) n + c n = 0...............(iv)

which is a homogeneous equation of second degree in x and y and hence always


represent a pair of straight lines passing through the origin and the point of intersection
of line (i) and curve (ii).
Example 10:
Show the lines joining the origin to the point of intersection of the line x + y = 1 with
the curve 4x2 + 4y2 + 4x – 2y –5 = 0 are at right angles to each other.

248 | Pioneer Mathematics Grade XI (Approved by CDC Nepal)


Solution:
Y
The given equation of straight line AB is, C
x + y = 1............. (i) A
L
Given equation of the curve, Q is
4x2+4y2+4x–2y –5 = 0..............(ii) B
Now, making equation (ii) homogeneous with the
help of equation (i) X' X
O
i.e. or 4x2+4y2+4x×1–2y×1–5×12 = 0
Y'
4x2+4y2+(4x–2y)(x+y)–5(x+y)2=0
or, 4x2+4y2+4x2+2xy–2y2–5x2–10xy–5y2 = 0
or, 3x2 – 8xy –3y2 = 0..............(iii)
Equation (iii) is the homogeneous equation of 2 nd degree representing a pair of straight
lines OA and OB and passing through the intersection of line (i) and curve (ii) and
through the origin.
Again, comparing equation (iii) with ax2 + 2hxy + by2 = 0, we get
a = 3, b = –3 and h = –4
The two lines given by (iii) are at right angles if a + b = 0 i.e. if 3–3 = 0
i.e. if, 0=0 which is true. Hence, the lines OA and OB are at right angles.
Example 11:
Determine the equations of straight lines represented by the equation.
2x2 + 7xy + 3y2 –4x –7y + 2=0
Solution:
Here, given equation is
2x2 + 7xy + 3y2 –4x –7y + 2 = 0
or, 2x2 + (7y – 4)x + 3y2 – 7y + 2 = 0 ................(i)
Equation (i) is quadratic in x, so by solving for x, we have,
–(7y –4)  (7y – 4)2 –4.2(3y2 – 7y + 2)
x= 2.2
or, 4x = –(7y –4)  49y2 –56y + 16 – 24y2 + 56y –16
or, 4x = – (7y –4)  25y2
or, 4x = –(7y –4)  5y
Taking +ve sign Taking –ve sign
4x = – (7y –4) + 5y 4x = –7y + 4 –5y
or, 4x = –7y + 4 + 5y or, 4x + 12y–4 = 0
or, 4x = –2y + 4 or, x + 3y –1 = 0...........(iii)
or, 4x + 2y –4 = 0
or, 2x + y –2 = 0.........................(ii)
Equation (ii) and (iii) are the required equations of the lines given by equation (i)

(Approved by CDC Nepal) Analytical Geometry | 249


Example 12:
If the line pair x2 – 2pxy –y2 = 0 and x2 – 2qxy – y2 = 0 be such that each pair bisects the
angles between the other pair, prove that pq = –1.
Solution:
Given equation of line pairs
x2 – 2pxy – y2 = 0...................(i)
and x2 – 2qxy – y2 = 0 ......................(ii)
Now, equations of bisectors of the angles between the lines given by (i)
h(x2 – y)2) = (a – b) xy
or, –p(x2 – y2) = [1 – (–1)] xy
or, –px2 + py2 = 2xy
or, px2 + 2xy – py2 = 0..............(iii)
But, by question the bisectors (iii) are the same lines given by (ii).
i.e. x2 – 2qxy – y2 = 0 and px2 + 2xy –py2 = 0 represent the same line pair
1 –2q –1
So, p = 2 = –p [∵ The coeff. of correspending terms are proportional]

1 1
⇒ p = –q = p
1
⇒ p = –q
⇒ pq = –1 proved.
Example 13:
Find the equations of the pair of straight lines joining to the intersection of the straight
lines y = mx + c and the curve x2 + y2 = a2. Prove that they are at right angles if
2c2 = a2(1 + m2).
Y P
Solution: C
A
Here, equation of given straight line is
L
y = mx + c Q
or, y – mx = c B
y – mx
or, = 1.........(i) X' X
c O
and, given equation of a curve is Y'
x 2 + y2 = a 2
or, x2 + y2 – a2 = 0...........(ii)
Now, making equation (ii) homogeneous with the help of line (i)
i.e. x2+y2–a2×l2 = 0
(y – mx)2
x 2 + y 2 – a2 c =0

or, c2 (x2 + y2) – a2(y – mx)2 = 0

250 | Pioneer Mathematics Grade XI (Approved by CDC Nepal)


or, c2(x2 + y2) – a2(y2 – 2mxy + m2x2) = 0
or, c2x2 – a2y2 + 2a2mxy + c2y2 – a2m2x2 = 0
or, c2x2 – a2m2x2 + 2a2mxy + c2y2 – a2y2 = 0
or, x2(c2 – a2m2) + 2a2mxy + y2(c2 – a2) = 0..........(iii)
Equation (iii) is the homogeneous equation of second degree that represent the pair of
lines passing through the intersection of the line (i) and curve (ii).
Now, the lines given by (iii) are at right angles if Coefficient of x2 + coefficient of y2 = 0
or, c2 – a2m2 + c2 – a2 = 0 or, 2c2 = a2 + a2m2
or, 2c2 = a2(1 + m2) which is the required condition.
Example 14:
Find the condition that one of the lines given by ax 2 + 2hxy + by2 = 0 may be
perpendicular to one of the lines given by a'x2 + 2h'xy + b'y2 = 0
Solution:
Here, given equations of line pairs are
ax2 + 2hxy +| by2 = 0..........(i)
and a'x2 + 2h'xy + b'y2 = 0...........(ii)
Let y = mx be equation of one of the lines given by (i), them
ax2 + 2hx.mx + bm2x2 = 0
or, bm2 + 2hm + a = 0.................(iii)
Now, equation of line perpendicular to y = mx and through the origin is
1
y=–mx  (m1m2 = –1) which is one of the lines given by (ii)

1 1
So, a'x2 + 2h'x (–m x) + b'm2 x2 = 0

or, a'm2 – 2h'm + b' = 0..........(iv)


Now, solving equations (iii) and (iv) by the rule of cross multiplication
m2 m 1
2(hb' + h'a) = aa' – bb' = –2(bh' + a'h)
2(hb' + h'a)
From first and second ratio's, m = aa' – bb' ............(v)
aa' – bb'
From second and third ratio's, m = – 2(bh' + a'h) ...........(vi)

Now, from (v) and (vi)


2(hb' + h'a) aa' – bb'
aa' – bb' = –2(bh' + a'h)
or, (aa' – bb')2 = –4(bh' + a'h) (hb' + h'a)
or, (aa' – bb')2 + 4(bh' + a'h) (hb' + h'a) = 0 which is the required equation.

(Approved by CDC Nepal) Analytical Geometry | 251


Example 15:
Prove that the product of the perpendiculars drawn from the point (x 1 ,y1) on the lines
ax 2 + 2hx1y1 + by12
represented by ax2 +2hxy + by2 = 0 is 1
(a – b)2 + 4h2
Solution:
Here given equation of line pair is
ax2 + 2hxy + by2 = 0...........(i)
Let, the of the straight lines represented by equation (i) be
y = m1x ..............(ii) and y = m2x............(iii)
2h a
Then, m1 + m2 = – b and m1.m2 = b …………….. (iv)

m1x1 – y1
Now, perpendicular distance of the line (ii) from the point (x 1,y1) is P1 = 
m12 + 1
m2x1 – y1
and perpendicular distance of the line (iii) from the point(x1,y1) is P2 = 
m22 + 1
Now, Product of the perpendiculars
m1x1 – y1    m2x1 – y1 
P1P2 = 
 m12 + 1  m22 + 1
(m1x1 – y1) (m2x1 – y1)
=
(m12 + 1) (m22 + 1)
m1m2x12 – (m1 + m2) x1y1 + y12
=
m12m22 + (m12 + m22) + 1
Substituting the values from above (*), we get
a 2 2h 2
bx1 + b x1y1 + y1
P1P2 =
(m1m1)2 + (m1 + m2)2 – 2m1m2 + 1
a 2 2h 2
bx1 + b x1y1 + y1
= 2 2
 a + –2h – 2. a + 1
b  b  b
ax12 + 2hx1y1 + by12
=
a2 + 4h2 – 2ab + b2
ax12 + 2hx1y1 + by12
∴ P1P2 =
(a – b)2 + 4h2

252 | Pioneer Mathematics Grade XI (Approved by CDC Nepal)


EXERCISE 3.2
1. Find a single equation representing each of the line pair
(a) x + y = 1 and x + 3y = 2 (b) y = 2x + 3 and 3x = 1 – y
(c) x = 4y and x = y (d) 3x – 2y = 0 and x – y = 0
2. Find the separate equation of the lines represented by the following equations.
(a) xy – 3y2 = 0 (b) x2 – 5xy + 4y2 = 0
(c) 2x2 – xy – y2 – 3x – 6y – 9 = 0 (d) y2 – 2xytan – x2 = 0
(e) x2 – 6xy sec + 9y2 = 0
3. (a) Find the separate equations and the point of intersection of the lines represented by
6x2 + 5xy –4y2 + 7x + 13y –3 = 0
(b) Determine the equations of the lines represented by x 2+6xy+9y2+4x+12y–5=0.
Prove that the lines are parallel.
4. Find the angle between the following line pair.
(a) 2x2 + 7xy + y2 = 0 (b) 7x2 + 8xy + y2 = 0
(c) x2 – 2xycot – y2 = 0 (d) x2 + 2xysec + y2 = 0
(e) x2 – 5xy + 4y2 + 3x –4 = 0 (f) 2x2 + 4xy + 2y2 + x + y –3 = 0
5. Find the equations of the bisectors of the angles between the line pair represented by the
following equations.
(a) 2x2 – 6xy – y2 = 0 (b) 3x2 – 15xy + 2y2 = 0
(c) ax2 + 2hxy + by2 + k(x2 + y2) = 0 (d) x2 + 2xysec + y2 = 0
6. Verify whether the each of the following equations represent a line pair.
(a) 12x2 – 5xy –3y2 + 3x + y= 0
(b) 2x2 + 7xy + 3y2 + 8x + 14y + 8 = 0
(c) 2x2 + 7xy + 3y2 –4x –7y + 2 = 0
(d) 3x2 + 10xy + 8y2 + 14x + 22y + 7 = 0
7. For what value of k will the following equations represent a pair of lines?
(a) 12x2 – 10xy + 2y2 + 11x – 5y + k = 0
(b) x2 + kxy + 2y2 + 3x + 5y + 2 = 0
(c) 2x2 + xy – y2 + kx + 6y –9 = 0
8. For what value of k the equations
(a) 4x2 –15xy + ky2 = 0 represents a pair of perpendicular lines.
(b) 3x2 – 4kxy + 5y2 = 0 represents a pair of coincident lines.

(Approved by CDC Nepal) Analytical Geometry | 253


9. Find the single equation of the lines through the origin and perpendicular to the lines
represented by ax2 + 2hxy + by2 = 0
10. If the equation ax2 + 2hxy + by2 + 2gx + 2fy + c = 0 represent a pair of parallel lines,
prove that

a h g g2 – ac
(a) h = b = f (b) The distance between them is 2 a2 + ab

11. Determine the equations of straight lines through the point (–1,2) which are
perpendicular to the straight lines 2x2 – 5xy –12y2 + 5x + 13y –3 = 0
12. Find the equation of straight lines passing through the origin and perpendicular to the
lines x2 + 2xy –3y2 = 0.
13. Find the condition that one of the lines ax 2 + 2hxy + by2 = 0 may be perpendicular to
one of the lines given by a1x2 + 2h1xy + b1y2 = 0
14. Find the condition that one of the lines ax2 + 2hxy + by2 = 0 may coincide with one of
the lines.a'x2 + 2h'xy + b'y2 = 0
15. (a) Find the equations of straight lines joining the origin to the point of intersection of
the line 3x –2y = 1 and the curve 3x2 + 5xy –3y2 + 2x + 3y =– 0. Also, prove that
the lines are at right angles.
(b) Find the value of p so that the lines joining the origin to the point of intersection of
the lines y – x = p and the curve x2 + y2 + 4x –6y –36 = 0 may be at right angles.
(c) Prove that the straight lines joining the origin to the point of intersection of the line
x y 2 2 2 1 1 2
a + b = 1 and the curve x + y = c are at right angles if a2 + b2 = c2
(d) Find the angle between the lines joining the origin to the points of intersection of the
line 3x – y + 2 = 0 and the curve x2 + 2xy + 3y2 + 4x + 8y –11 = 0.
16. Prove that the two straight lines (x2 + y2) sin2 = (xcos – ysin)2 include an angle 2.
17. Prove that the two straight lines x2(tan2 + cos2) – 2xytan + y2sin2 = 0 make with x
axis angles such that the difference of their tangents is 2.
18. If the two pairs of lines given by x2 – 2pxy –3y2 = 0 and x2 – 2qxy – y2 = 0 be such that
each pair bisects the angles between the other pair, prove that pq + 2 = 0.

ANSWERS
1. a) x2 + 4xy + 3y2 –3x –5y + 2 = 0 b) 6x2 – xy – y2 + 7x + 4y + 3 = 0
c) x2 – 5xy + 4y2 = 0 d) 3x2 – 5xy + 2y2 = 0
2. a) y = 0, x – 3y = 0 b) x – 4y = 0, x – y = 0
c) x – y –3 = 0, 2x + y + 3 = 0 d) y=(tan + sec)x, y = (tan – sec)x
e) x cos – 3y(1 sin) = 0

254 | Pioneer Mathematics Grade XI (Approved by CDC Nepal)


3. a) 2x – y + 3=0, 3x + 4y –1 = 0, (–1,1) b) x + 3y + 5 = 0, x + 3y –1 = 0

4.
 41
a) tan–1  3
3
b) tan–1  4 c) 90º d)  
   
3
e) tan–1  5 f) 0º.
 
5. a) 3x2 + 3xy – 3y2 = 0 b) 15x2 + 2xy – 15y2 = 0
c) hx2 – (a–b)xy – hy2 =0 d) x = y
7. a) 2 b) 3 or 9/2 c) – 3
15
8. a) –4 b) 2
9. ay2 – 2hxy + bx2 = 0
11. 12x2 – 5xy – 2y2 + 34x + 3y + 14 = 0
12. 3x2 + 2xy – y2 = 0
13. (aa1 – bb1)2 + 4(bh1 + a1h). (b1h + ah1) = 0
14. 4(a'h – ah')(bh' – b'h) = (ab' – a'b)2

15. a) 9x2 + 10xy – 9y2 = 0 b) 9 or – 4


 2 2
d) tan–1  3
 

Objectives Questions
Circle the correct Answers
1. Locus of the equation xy - 2x - 3y + 6 = 0 is
a. a pair of horizontal lines b. a pair of vertical lines
c. a pair of oblique lines d. a pair of perpendicular lines
2 2
2. If the equation x + y +2gx+2fy+1 = 0 represents a pair ot lines then
1 1
a. f2-g2 = b. g2-f2 = 1 c. f2+g2 = 1 d. f2+g2 =
2 2
3. Pair of straight lines perpendicular to each other is represented by
a. 2x2 = 2y (2x+y) b. x2+y2+3=0 c. 2x2=y(2x+y) d. x2=2(x-y)
4. If  is the angle between the lines given by x2-2pxy+y2 =0 then
a. cos = p b. sec = p c. tan = p d. cot=p
2 2
5. The distance between the parallel lines 9x - 6xy + y + 8x - 6y + 8 = 0 is
2 1 4
a. b. c. d. None
10 10 10
6. The points of intersection of the lines 6x2 - 3xy - 3y2 + 15x + 2y + 6 = 0 is
a. (-1,1) b. (1,-1) c. (3,-3) d. (-3,3)
7. The product of the length of the perpendiculars from the point (1,2) to the lines 2x 2+5xy-4y2 = 0 is
3 5 2 4
a. b. c. d.
17 49 31 61

(Approved by CDC Nepal) Analytical Geometry | 255


8. The lines joining the points of intersection of the lin x+y=1 and the curve x2+y2 - 2y +  = 0 to the
origin are perpendicular. Then value of  =
1 1 1
a. b. - c. 0 d.
2 2 2
9. If the slope of one of the lines given by a2x2+2hxy+b2y2 = 0 is
2 3
a. b. 2a2b2 c. 4ah d. ab
3 2
10. If the equation 12x2 + 7xy - py2 - 18 x + qy + 6 = 0 represents a pare of perpendicular lines, then
a. P = 12, q = 1 b. p = 1, q = 12 c. p = -1, q = 12 d. p = 1, 1 = -12
11. If 4ab = 3h2, then ratio of slopes of the lines represented by the equation ax2 + 2hxy + by2 = 0 will be
a. 1:3 b. 2:1 c. 2 :1 d. 3 :1
2 2
12. The sum of slope of the lines represented by 4x +2hxy-7y = 0 is equal to the product of slopes then
h is equal to (IOE)
a. -4 b. 4 c. -6 d. -2

Answers:

1 d 2 c 3 a 4 b 5 a
6 b 7 d 8 c 9 a 10 a
11 a 12 d

256 | Pioneer Mathematics Grade XI (Approved by CDC Nepal)


3.3

Learning Objective

After the completion of this chapter, students will be able to understand


 solve the problems related to condition of tangency of a line at a point to the circle.
 find the equations of tangent and normal to a circle at given point.

(Approved by CDC Nepal) Analytical Geometry | 257


Review of circle

The word 'circle' was derived from the Greek work 'Kirkos' which means to turn or bend. The
circle has been known since before the beginning of the recorded history.
In 1700 BC, the Rhind Papyrus gave a method to find the area of a circular field. He also found
the ratio of circumference and diameter of a circle is always same for all circles. The ratio is
termed as π (Pi) and the result corresponds to 256/81 (3.16049…) as an approximate value of
π. In 1880 BC, Lindemann proved that π is transcendental effectively setting the millennia.
The circle is the basis for the wheel, which with related inventions such as gears, makes
much of modern machinery possible. In mathematics, the study of the circle has helped
inspire the development of geometry, astronomy calculus. Early science, particularly
geometry and astrology and astronomy was connected to the divine for most medieval
scholars and many believed that there was something intrinsically 'divine' or 'perfect' that
could be found in circles. P (x, y)

Equation of a Circle
A circle is defined as the locus of a point which moves in such C

a way that its distance from a fixed point is always constant.

The constant (fixed) distance is called the radius denoted by r Y


and the fixed point is called the center of the circle. The ratio
of the circumference of a circle to its diameter is same for all P(x, y)

circle and the ratio is called π (Pi).


(i) Central form
C(h, k)

A circle with centre at a point (h,k) and of radius r is given by (x – X' X


h)2 + (y – k)2 = r2 O

Y'
Whose the centre is at (–g, – f) and radius is = g2 + f2 – c
Remarks:
(i) If g2 + f2 – c > 0, the radius is real and hence the equation gives a real circle.
(ii) If g2 + f2 – c = 0, the radius is zero and hence the circle in this case is a point circle.
(iii) g2 + f2 – c < 0, the radius becomes imaginary and hence the equation represents a
circle with a real centre and an imaginary radius.
(iv) The parametric equation of a circle x2 + y2 = a2 is x = a cos and y = a sin.
(v) The parametric equation of a circle (x – h)2 + (y – k)2 = a2 is
x = h + acos and y = k + asin.
Y
Some Special Cases
(i) Circle touching the x axis
P(x, y)
Let (h, k) be the centre of the circle and r be the radius.
When the circle touches the x axis then r = k (h, k)

Then equation of a circle touching x-axis is r


2 2 2 2 2 2
(x – h) + (y – k) = k or, (x – h) + (y – r) = r X'
O
h
X

Y'

258 | Pioneer Mathematics Grade XI (Approved by CDC Nepal)


(ii) Circle touching y-axis
Let (h,k) be the centre of the circle and r be the radius.
when the circle touches the y axis
Then r = h Y
Then equation of a circle toucting y - axis is
(x – h)2 + (y–k)2 = h2 or, (x –h)2 + (y – k)2 = r2
(iii) Circle touching both axes:
h
Let (h, k) be the centre of the circle and r be the radius. (h, k)
k
When the circle touches both axes, then
X' X
h = k = r........(ii) O
Then, equation of the circle touching both axes is Y'
(x – h)2 + (y – k)2 = h2 or, (x –h)2 + (y – k)2 = k2 Y
or, (x – r)2 + (y – r)2 = r2 B(x 2 ,y2 )

(iv) Circle in Diameter form


If (x1,y1) and (x2,y2) be the end points of a diameter of a
circle, then its equation is
(x –x1) (x – x2) + (y – y1) (y – y2) = 0. A(x 1 ,y1 )
X' X
Tangent and Normal of a Circle O
A Point and a Circle Y'
Let, x2 + y2 + 2gx + 2fy + C = 0..........(i) be the equation of a circle with the centre at C(–g,–f)
and radius (r) = g2 + f2 – c P(x1 , y1)
Let P(x1,y1) be any point and let us denote equation (i)
by F(x, y) such that
F(x,y) = x2 + y2 + 2gx + 2fy + c, then r
2 2
F(x1,y1) = x + y + 2gx1 + 2fy1 + c
1 1

= x12 + 2gx1 + g2 + y12 + 2fy1 + f2 – g2 – f2 + c C(-g, -f)


2 2 2 2
= (x1 + g) + (y1 + f) – (g + f – c)
= CP2 – r2
 F(x1,y1) = CP2 – r2...........(ii)
Thus,
(i) If F(x1,y1) = CP2 – r2 = 0, then P lies on the circle.
(ii) If F(x1,y1) = CP2 – r2 < 0, then P lies inside the circle.
(iii) If F(x1,y1) = CP2 – r2 > 0, then P lies outside the circle.
Remarks:
A point (x1,y1) lie inside the circle x2 + y2 = a2 if x12 + y12 – a2 < 0
A point (x1,y1) lie on the circle x2 + y2 = a2 if x12 + y12 – a2 = 0
A point (x1,y1) lie on the circle outside x2 + y2 = a2 if x12 + y12 – a2 > 0

(Approved by CDC Nepal) Analytical Geometry | 259


Intercepts made by a circle
Let the given circle be x2 + y2 + 2gx + 2fy + c = 0, Then
(i) length of x–intercept is = 2 g2 – c
(ii) length of y–intercept is = 2 f2 – c
(iii) the circle touches x-axis if g2 = c
(iv) the circle touches y–axis if f2 = c
(v) the circle touches both axes if g2 = f2 = c
Example 1:
Determine whether the point (–3,1) lies on or, inside or outside the circle x2 + y2 = 9.
Solution:
Here, given circle is
x2 + y2 –9 = 0...........(i) and given point is (–3, 1)
Substituting ( –3,1) on LHS,
(–3)2 + (1)2 –9 = 9+1 –9 =1 >0
So, the point (–3, 1) lies outside the circle.
A Line and a Circle
Let the equation of a circle x2 + y2 = a2 .......(i)
and the equation of a line y = mx + c............(ii)
Now, solving equation (i) and (ii), we get
x2 + (mx + c)2 = a2
or, x2 + m2x2 + 2mcx + c2 = a2
or, x2 (1 + m2) + 2mcx + c2 – a2 = 0............(iii)
This is quadratic equation in x which has two values.
Discriminant of equation (iii) is
D = b2 – 4ac
= (2mc)2 – 4(1 + m2) (c2 – a2) [∵ D = b2 – 4ac]
2 2 2 2 2 2 2 2
= 4m c – 4c + 4a – 4m c + 4a m
= 4(a2m2 – c2 + a2)
= 4[a2 (1 + m2) – c2]
There may arise three cases:
Case 1: The roots of equation (iii) may real and distinct In this case the line cuts the circle at
two distinct points and line RS is called secant
Case 2: The roots of equation (iii) and real and equal. In this case the line meets the circle at
two coincident points and line RS is called tangent to the circle.
Case 3: The roots of equation (iii) are imaginary. In this case the line doesn't intersect the
circle.
Hence,
(i) the line (i) will be tangent to the circle if a 2(1 + m2) = c2
(ii) The line (i) will be secant to the circle if a 2(1 + m2) > c2
(iii) The line (i) neither intersect nor touch the circle if a2(1 + m2) < c2.

260 | Pioneer Mathematics Grade XI (Approved by CDC Nepal)


Tangents and Normals to any Curve
Let P be a point on a curve. Take another point Q on the
curve. If PQ is joined, a secant line PQ is formed. As the
point Q moves along the curve to the point P (i.e. QP) R
then the secant line PQ will tend to a definite line PR
through P. This new line PR is called the tangent to the
P
curve at P and the point P is called the point of contact.
Any line that passes through the point of contact and
perpendicular to tangent is called the normal.

Tangent and Normal to a Circle


A straight line is said to be a tangent to a circle if it touches
C
the circle at a single point P. In other words, a straight line is
said to be a tangent to a circle if it meets the circle at two
coincident points. P
A straight line which is perpendicular to the tangent to the
circle at a point is called normal to a circle at a point.
Remarks:
(i) The point of intersection of tangent and normal is called point of contact.
(ii) A normal line always passes through the centre of the circle.
(iii) The length of perpendicular from the centre of the circle to the tangent of a circle is
called the radius of the circle.
Equation of Tangent
(i) To find the equation of tangent to the circle x 2 + y2 = a2 at the point (x1,y1) on the circle.
Let P(x1,y1) be a given point on the circle x2 + y2 = a2. Take another point Q(x2,y2) on the
same circle near to P.
Since P and Q lie on the circle x2 + y2 = a2.
Then, x12 + y12 = a2 ............ (i) Q (x2,y2)
2 2 2
and x + y2 = a ................(ii)
2

Subtracting (i) from (ii) we get


(x22 + y22) – (x12 + y12) = 0
P(x1,y1)
or, x22 + y22 – x12 – y12 = 0
or, (x22 – x12) + (y22 – y12) = 0
or, (y22 – y12) = – (x22 – x12)
or, (y2 – y1) (y2 + y1) = – (x2 – x1) (x2 + x1)
y –y x +x
or, x2 – x1 = – y2 + y1 ................(iii)
2 1 2 1

Now, equation of the secant PQ is


y –y
y – y1 = x2 – x1 (x – x1) .................... (iv) (Using two points form)
2 1

(Approved by CDC Nepal) Analytical Geometry | 261


x +x
or, y – y1 = – y2 + y1 (x – x1) .............(v) (using (iii))
2 1

By definition of tangent, the secant line PQ becomes the tangent at point P when the
point Q tends to P (QP) i.e. when x2 x1 and y2y1.
Therefore, equation (v) becomes
x +x
y – y1 = – y1 + y1 (x – x1)
1 1

2x
or, y – y1 = – 2y1 (x – x1)
1

or, yy1 – y = – xx1 + x12


1
2

or, xx1 + yy1 = x12 + y12


or, xx1 + yy1 = a2 [using (i)]
Thus, the equation of the tangent to the circle x 2+y2=a2 at the point (x1,y1) is xx1+yy1=a2
Alternative method – I
Let P(x1,y1) be any point on the circle x2 + y2 = a2........... (i)
y
Here, the slope of OP = x1
1

As the tangent at P is perpendicular to the radius


x
 The slope of the tangent at P = – y1
1

 The equation of the tangent at P is (0,0)


x
y – y1 = – y1 (x – x1)
1

or, yy1 – y12 = – xx1 + x12 P(x1,y1)


2 2
or, xx1 + yy1 = x + y 1 1

or, xx1 + yy1 = a2 (using (i))


Which is the required equation of the tangent to the circle (i) at point (x 1,y1) on it.
Alternative method – II (with the help of derivative)
Given equation of the circle is
x2 + y2 = a2.........(i)
Now, differentiating (i) with respect to x, we get
dy
2x + 2ydx = 0

dy
or, 2y dx = –2x P(x1,y1)

dy 2x
or, dx = – 2y

dy x
or, dx = – y

262 | Pioneer Mathematics Grade XI (Approved by CDC Nepal)


Let , m be the slope of the tangent to the circle (i) at P(x 1,y1).
dy x1
Then m = dx
  (x1,y1) = – y1
 The equation of the tangent at P (x1,y1) is y – y1 = m(x – x1)
x
or, y – y1 = – y1 (x – x1)
1

or, yy1 – y = – xx1 + x12


1
2

or, xx1 + yy1 = x12 + y12


or, xx1 + yy1 = a2 (point P(x1,y1) lies on the circle so x12 + y12 = a2)
(ii) To find the equation of the tangent to the circle x 2 + y2 + 2gx + 2fy + c = 0 at a point
(x1,y1) on it
Let P(x1,y1) be a point on the circle
x2 + y2+2gx + 2fy + c = 0....(i)
Take another point Q (x 2,y2) on the circle near to P.
Since the points P(x1,y1) and Q(x2,y2) lie on the given circle (i)
x12 + y12 + 2gx1 + 2fy1 + c = 0.............(ii)
and x22 + y22 + 2gx2 + 2fy2 + c = 0.............(iii)
Subtracting (i) from (iii), we get
x22 – x12 + y22 – y12 + 2g(x2 – x1) + 2f(y2 – y1) = 0
or, (x2 – x1) (x2 + x1) + (y2 – y1) (y2 + y1) + 2g(x2 – x1) + 2f(y2 – y1) = 0
or, (x2 – x1) (x2 + x1 + 2g) +(y2 – y1) (y2 + y1 + 2f) = 0
or, (y2 – y1) (y2 + y1 + 2f) = – (x2 – x1) (x2 + x1 + 2g)
y –y x + x + 2g
or, x2 – x1 = – y2 + y1 + 2f ..........(iv)
2 1 2 1

Now, equation of the secant PQ is


y –y
y – y1 = x2 – x1 (x – x1)
2 1

x + x + 2g
or, y–y1 = –  y1 + y2 + 2f  (x – x1) (using (iv)...] (v)
 1 2 
By the definition of tangent, the secant line PQ becomes the tangent at point P when the
point Q tends to P (i.e. Q  P) i.e. when x2 x1 and y2 y1.
Then, equation (v) becomes
x + x + 2g
y – y1 = –  y1 + y1 + 2f  (x – x1)
 1 1 
2(x + g)
or, y – y1 = – 2(y1 + f) (x – x1)
1

or, (y – y1) (y1 + f) = – (x1 + g) (x – x1)


or, yy1 + yf – y12 – y1f = – (xx1 – x12 + gx – gx1)
or, xx1 + yy1 + gx + fy = x12 + y12 + gx1 + fy1

(Approved by CDC Nepal) Analytical Geometry | 263


Adding gx1 + fy1 + c to both sides, we get
xx1 + yy1 + g(x + x1) + f(y + y1) + c = x12 + y12 + 2gx1 + 2fy1 + c
i.e. xx1 + yy1 + g(x + x1) + f(y + y1) + c = 0 (Using (iii))
which is the required equation of the tangent to the circle (i) at point (x 1,y1) on it.
Alternative method
Let P(x1,y1) be the given point on the given circle.
x2 + y2 + 2gx + 2fy + C = 0...........(i)
Then x12 + y12 + 2gx1 + 2fy1 + C = 0........(ii)
Let C denote its centre (–g, –f). Then
y – (– f) y +f  slope = y2 – y1
Slope of CP = x1 – (–g) m = x1 + g
1 1  x2 – x1
As the tangent to the circle (i) at the point P is perpendicular to CP, so
1 x +g
Slope of the tangent at P = –m = – y1 + f
1

 Equation of tangent to the circle (i) at point P(x 1,y1) is


x +g
y – y1 = – y1 + f (x – x1)
1

or, (y – y1) (y1 + f) = – (x1 + g) (x – x1)


or, (x – x1) (x1 + g) + (y – y1) (y1 + f) = 0
or, xx1 + yy1 + gx + fy = x12 + y12 + gx1 + fy1
or, xx1 + yy1 + gx + fy = – gx1 – fy1 – c
or, xx1 + yy1 + g(x + x1) + f(y + y1) + c = 0
which is the required equation of the tangent to the circle (i) at point P(x 1,y1) on it.
Remark: The equation of the tangent to a circle (i) at the point (x1,y1) on it can be written down
by substituting xx1 for x2, yy1 for y2, (x + x1) for 2x and (y + y1) for 2y.
Condition of Tangency
Find the condition that the line y = mx + c will be tangent to the circle x2 + y2 = a2.
Given equation of the circle is x2 + y2 = a2..........(i)
and given equation of the straight line is y = mx + c.........(ii)
Solving (i) and (ii).
x2 + (mx + c)2 = a2
or, x2 + m2x2 + 2mcx + c2 – a2 = 0
or, (1 + m2)x2 + (2mc)x + (c2 – a2) = 0............(iii)
Which is a quadratic equation in x. The line (ii) will touch the circle if the two values of x
obtained from the above quadratic equation are equal.
i.e. if discriminant of the equation (iii) is to zero.
i.e. if (2mc)2 – 4(1 + m2) (c2 – a2) = 0
i.e. if 4m2c2 – 4(c2 – a2 + m2c2 – m2a2) = 0

264 | Pioneer Mathematics Grade XI (Approved by CDC Nepal)


i.e. if m2c2 – c2 + a2 – m2c2 + m2a2 = 0
i.e. if c2 = a2 + m2a2
i.e. if c2 = a2(1 + m2)
i.e. if c =  a 1 + m2
which is the required condition of tangency.
Alternative method
Given equation of the circle is
x2 + y2 = a2.........(i)
and given equation of the line is y = mx + c.........(ii)
Here, centre of the circle is C(0,0) and radius of the circle is
CP = a
The line PQ is a tangent to the circle (i) if the length of perpendicular from the centre of the
circle to the line is equal to the radius of the circle.
m.0 – 0 + c
i.e. if. a = 
m2 + (–1)2
c
i.e. if, a = 
m2 + 1
i.e. f, a m2 + 1 =  c
or, c =  a 1 + m2 which is the required condition of tangency.
Remarks:
(i) The straight lines y = mx  a 1 + m2 are always tangent to the circle x2 + y2 = a2.
(ii) y = mx  a 1 + m2 is also known as equation of tangent in terms of its gradient.
Equation of Normal
(i) Find the equation of a normal to the circle x2 + y2 = a2 at P(x1,y1) on the circle.
Given equation of the circle is x2 + y2 = a2........(i)
The equation of a tangent to the circle (i) at p(x1,y) is xx1 + yy1 = a2
x
Here, slope of the tangent = – y1
1

Since normal passes through the centre and perpendicular to the


tangent of the circle.
y
Slope of the normal = x1
1

Equation of the normal to the circle (i) at P(x1,y1) is


y
y – y1 = x1 (x – x1)
1

or, x1y – y1x1 = xy1 – x1y1


or, x1y = y1x

(Approved by CDC Nepal) Analytical Geometry | 265


(ii) To find the equation of the normal to the circle x 2 + y2 + 2gx + 2fy + c = 0 at point
P(x1,y1) on the circle.
Given equation of the circle is x2 + y2 + 2gx + 2fy + c = 0...........(i)
We have the equation of the tangent to the circle (i) at point P(x1,y1) is
xx1 + yy1 + g(x + x1) + f(y + y1) + c = 0
or, xx1 + yy1 + gx + gx1 + fy + fy1 + c = 0
or, (x1 + g)x + (y1 + f) y + (gx1 + fy1 + c) = 0........(ii)
Since normal perpendicular to the tangent. So, equation of the line perpendicular to line (ii) is
(y1 + f)x – (x1 + g)y + K = 0........(iii)
If it passes through P((x1,y1) then
(y1 + f) x1 – (x1 + g) y1 + k = 0
or, x1y1 + x1f – x1y1 – gy1 + k = 0
or, x1f – gy1 + k = 0
or, k = gy1 – fx1
There, Equation (iii) becomes
(y1 + f)x – (x1 + g) y + (gy1 – fx1) = 0
or, (y1 + f)x – (x1 + g) y = fx1 – gy1
which is the required equation of the normal to the circle (i) at the point (x 1,y1).
Tangent from an External Point
Show that two tangents can be drawn from an external point
to a circle.
Proof: To prove the thorem, we consider the equation of the
circle x2 + y2 = a2.........(i)
Let P((x1,y1) be an external point.
We have, the equation of the tangent to the circle (i) having
slope m is y = mx + a 1 + m2
If it passes through the point (x 1,y1), then y1 = mx1 + a 1 + m2
or, (y1 – mx1) = a 1 +m2
or, (y1 – mx1)2 = a2(1 + m2) (squaring both sides)
or, y12 – 2mx1y1 + m2x12 = a2 + a2m2
or, y12 – 2mx1y1 + m2x12 – a2 – a2m2 = 0
or, (x12 – a2)m2 – 2mx1y1 – (y12 – a2) = 0......(ii)
This is a quadratic equation in m, which gives two values of m.
Thus, two tangents can be drawn from an external point to a circle, x2+y2=a2.
Similarly, we can prove the theoem for a circle x 2+y2+2gx+2fy+C = 0
Hence two tangents can be drawn from an external point to a circle.

266 | Pioneer Mathematics Grade XI (Approved by CDC Nepal)


Remarks:
The two tangents drawn will be real and distinct, coincident or imaginary according as the
given point lies outside, on or inside the circle.
Length of the length of the tangent from the point P(x 1,y1) to the circle
x2 + y2 + 2gx + 2fy + c = 0.
Given equation of the circle is x2 + y2 + 2gx + 2fy + c = 0
Let P(x1,y1) be an external point from which a tangent to the circle (i) is
drawn and T is a point of contact of tongent and circle then PT is called
the length of the tangent ment line. Here, centre of the circle (i) is
C (–g,–f) and radius of the circle (i) is CT = g2 + f2 – c .
Also, CP = (x1 (–g))2 + (y1 – (–f))2 = (x1 + g)2 + (y1 + f)2)
and CT = radius = g2 + f2 – c
As CTP is a right angled triangle,
PT2 = CP2 – CT2 [∵ p2=h2–b2]
= (x1 + g)2 + (y1 + f)2 – (g2 + f2 – c)
= x12 + 2gx1 + g2 + y12 + 2gy1 + f2 – g2 – f2 + c
= x12 + y12 + 2gx1 + 2fy1 + c
 PT = x12 + y12 + 2gx1 + 2fy1 + c
which gives the length of the tangent from an external point P(x 1,y1) to the circle (i).
Remarks:
(i) the length of the tangent from an external point (x1,y1) to the circle x2 + y2 = a2 is
x12 + y12 – a2
Example 1:
Find the equations of tangents and normals to each of the following circles:
(a) x2 + y2 = 36 at ( –6,0)
(b) x2 + y2 – 3x + 10y –15 = 0 at (4,–11)
Solution:
(a) We know that equation of the tangent to the circle x 2 + y2 = a2 at point (x1,y1) is
xx1 + yy1 = a2
 Equation of the tangent to the circle x2 + y2 = 36 at point (–6,0) is
x.(–6) + y.0 = 62
or, –6x = 36
or, x = –6
1
Here, the slope of the tangent = 0 . So, the slope of the normal = 0

 Equation of the normal at (–6, 0) is, y –0 = 0(x + 6) or, y = 0


(b) We know that equation of the tangent to the circle x 2 + y2 + 2gx + 2fy + c = 0 at point
(x1,y1) is xx1 + yy1 + g(x + x1) + f(y + y1) + c = 0
 Equation of the tangent to the circle x2 + y2 – 3x + 10y –15 = 0 at (4,–11) is

(Approved by CDC Nepal) Analytical Geometry | 267


–3
x.4 + y.–11 +  2  (x + 4) + 5(y – 11) + (–15) = 0
 
3
or, 4x –11y – 2 x –6 + 5y –55 –15 = 0

3
or, 4x – 6y – 2 x –76 = 0

or, 8x –12y –3x –152 = 0


or, 5x –12y –152 = 0
5 12
Here, the slope of the tangent = 12 . So, the slope of the normal = – 5
 Equation of the normal at (4,–11) is
–12
y – (–11) = 5 (x – 4)
–12
or, y + 11 = 5 (x –4)
or, 5y + 55 = –12x + 48
or, 12x + 5y + 7 = 0
Example 2:
Find the points in which the line y = 2x + 1 cuts the circle x 2 + y2 = 2. Also find the
length of the chord intercepted by the circle on the line.
Solution:
Given equation of the circle is
x2 + y2 = 2...........(i)
And given equation of the line is y = 2x + 1.......(ii)
Substituting the value of y from (ii) in (i) we get
x2 + (2x + 1)2 = 2
or, x2 + 4x2 + 4x + 1 = 2
or, 5x2 + 4x –1 = 0
or, 5x2 + 5x –x –1 = 0
or, 5x(x + 1) –1(x + 1) = 0
or, (5x –1) (x + 1) = 0
1
 x = 5 or x= –1

1 1 7
when x = 5 , y = 2.5 + 1 = 5
and when x = –1, y = 2.–1 + 1 = –1
1 7
Hence, the required co–ordinates of the points of intersection are 5 5 and (–1,–1)
 
2 2

Now, length of the chord PQ = 1 + 1 + 7 + 1 = 180 6


5  5  25 = 5 5
Remarks:
Length of the chord can also be calculated from the following formula
length of the chord = 2 (radius)2 – (distance of chord from centre)2

268 | Pioneer Mathematics Grade XI (Approved by CDC Nepal)


Example 3:
Prove that the tangent to the circle x2 + y2 = 5 at the point (1,–2) also touches the circle
x2 + y2 –8x + 6y + 20 = 0 and find the point of contact.
Solution:
Given equation of the circle is x2 + y2 = 5.........(i)
Equation of the tangent to the circle (i) at point (1,–2) is (3, –1)
x.1 + y.(–2) = 5
or, x –2y = 5..........(ii)
Given equation of another circle is
x2 + y2 –8x + 6y + 20 = 0........(iii)
Now, solving equation (ii) and (iii)
(2y + 5)2 + y2 – 8(2y + 5) + 6y + 20 = 0
or, 4y2 + 20y + 25 + y2 –16y –40 + 6y + 20 = 0
or, 5y2 + 10y + 5 = 0
or, 5(y2 + 2y + 1) = 0
or, y2 + 2y + 1 = 0
or, (y + 1)2 = 0
 y = –1,–1
when y = –1, x = 2.(–1) + 5 = 3
As the line (ii) meets the circle (iii) at coincident point (3,–1). So it touches the circle (iii)
i.e. Tangent to the circle (i) at point (1,–2) also touches the circle (iii) and the point of
contact is (3,–1).
Example 4:
Find the equations of the tangents to the circle x 2 + y2 – 4x + 6y –12 = 0 which are
perpendicular to the line 12x + 5y + 9 = 0
Solution:
Given equation of the circle is
x2 + y2 –4x + 6y –12 = 0............(i)
Comparing (i) with x2 + y2 + 2gx + 2fy + c = 0, we get
g = –2, f = 3, c = –12
 Centre of a circle = (–g, –f) = (2,–3)
and radius = g2 + f2 – c = 4 + 9 + 12 = 25 = 5
Any line perpendicular to 12x + 5y + 9 = 0 is 5x –12y + k = 0........(ii)
The line (ii) will be a tangent to the circle (i) if the length of perpendicular from the
centre of the circle to the line is be equal to the radius of the circle.
i.e. radius = length of perpendicular from (2,–3) to the line (i)
5.2 –12.(–3) + k
or, 5 = 
52 + (–12)2
10 + 36 + k
or, 5 = 
25 + 144
k + 46
or, 5 =  13
or, 65 =  (k + 46)

(Approved by CDC Nepal) Analytical Geometry | 269


Taking +ve sign 65 = k + 46 Taking – ve sign, 65 = –k –46
or, k = 65 –46 or, k = –65 –46
or, k = 19 or, k = –111
Hence, the required equation of tangents are 5x – 12y + 19 = 0 and 5x –12y –111 = 0
Example 5:
Find the equation of the tangents to the circle x 2 + y2 = 3 which make an angle of 60°
with the x–axis.
Solution:
Given equation of the circle is x2 + y2 = 3.........(i)
Comparing it with x2 + y2 = a2, we get a2 = 3 ⇒ a = 3
Let m be slope of the tangent which makes an angle 60° with the x–axis.
Then, m = tan60° = 3
No,w the equation of the tangents are
y = mx  a 1 + m2
or, y = 3 x  3 1 + 3
or, y = 3x  2 3
Example 6:
Find the value of k so that the line 2x–y+4k=0 touches the circle x2 + y2 –2x –2y –3 = 0.
Solution:
Here, the given equation of circle is
x2 + y2 –2x –2y –3 = 0..........(i) and the line is 2x–y+4k = 0........... (ii)
Comparing equation (i) with x2 + y2 + 2gx + 2fy + c = 0 we get
 Centre of the circle = (–g,–f) = (1,1)
and radius of the circle = g2 + f2 –c = 12 + (–1)2 – (–3) = 5
Since the line touches the circle so the length of the perpendicular from the centre (1,1)
to the line must be equal to the radius of the circle.
2.1 –1 + 4k
i.e. 5 = j
22 + (–1)2
1 + 4k
or, 5 =
5
or, 5 =  (1 + 4k)
Taking +ve sign and taking –ve sign
5 = 1 + 4k 5 = –1 –4k
 4k = 4  4k = 1– 5
 k=1  4k = –6
3
 k=–2

 k = 1 and –3/2
Remarks:
(i) If a line touches the circle or if a line is a tangent to the circle then.
Radius = length of perpendicular from the centre of the circle on the tangent line.
(ii) In case of standard equation of the circle either we can apply the condition of the
tangency c =  a 1 + m2 or we can apply remark (i).

270 | Pioneer Mathematics Grade XI (Approved by CDC Nepal)


Example 7:
Show that the tangents to the circle x2 + y2 = 100 at the points (6,8) and (8,–6) are
perpendicular to each other.
Solution:
The, given equation of the circle is x2 + y2 = 100 .......(i)
We know that the equation of the tangent to the circle x 2 + y2 = a2 at (x1,y1) is
xx1 + yy1 = a2
So, the equation of the tangent to the circle (i) at point (6,8) is
x.6 + y.8 = 100
or, 3x + 4y = 50..........(ii)
3
Here, Slope of the tangent (ii) is (m1) = – 4
Again, The equation of the tangent to the circle (i) at point (8,–6) is
x.8 + y.–6 = 100
or, 4x –3y = 50..........(iii)
4
Here, slope of the tangent (iii) is (m2) = 3
3 4
Here, m1 × m2 = –4 × 3 = –1,
The tangent (ii) and (iii) are perpendicular to each other.
Example 8:
If the line lx + my = 1 touches the circle x2 + y2 = a2, prove that the point (l,m) lies on
1
the circle of radius a .
Solution:
Given equation of the circle is x2 + y2 = a2..............(i)
and given equation of the line is lx + my = 1.......(ii)
As the line (ii) touches the circle (i), the length of perpendicular from the centre (0,0) is
equal to radius of the circle a.
l.0 + m.0 –1
i.e. a = 
l2 + m2
(–1)
or, a =  2
l + m2
1
or, a2 = l2 + m2 (Squaring both sides)
1
or, l2 + m2 = a2 .........(iii)
1
 The locus of (l,m) is x2 + y2 = a2
1 1
Which shows that, the point (l,m) lies on the circle x2 + y2 = a2 whose radius is a .
Alternative method
Given equation of the circle is
line is, x2 + y2 = a2...........(i)
and lx + my = 1
or, my = – lx + 1

(Approved by CDC Nepal) Analytical Geometry | 271


l 1
or, y = – m x + m ..........(ii)
If the line (ii) be a tangent to a given circle then by condition of tangency, we have
1 l2
m =a 1 + m2 [∵ C =  a 1+m2

1 m2 + l2
or, m =  a m2
1 a l2 + m2
or, m =  m
or, 1 =  a l2 + m2
or, 1 = a2 (l2 + m2) (squaring both sides)
1
or, l2 + m2 = a2
1
 The locus of (l,m) is x2 + y2 = a2
1 1
Which shows that, the point (l,m) lies on the circle x2 + y2 = a2 whose radius is a .

Example 9:
Prove that the two circles x2 + y2 + 2ax + c2 = 0 and x2 + y2 + 2by + c2 = 0 touch if
1 1 1
a2 + b2 = c2
Solution:
r1 r2
Given equation of the circles are
C1 C2
x2 + y2 + 2ax + c2 = 0........(i)
and x2 + y2 + 2by + c2 = 0.........(ii)
Let C1 and C2 be the centres and r1 and r2 be the radii of the circles (i) and (ii) respectively
C1 = (–a,0) and C2 = (0, –b)
also, r1 = a2 – c2 and r2 = b2 – c2
Two circles (i) and (ii) touch each other if the distance between their centres is equal to
sum or difference of their radaii.
i.e. if if C1C2 = r1 r2
i.e. if, (–a –0)2 + (0 + b)2 = a2 – c2  b2 – c2
i.e. if, a2 + b2 = a2 – c2  b2 – c2
[∵ a2 + b2 = a2 – c2 2 a2 – c2 . b2 – c2 + b2 – c2
i.e. if, 0 = –2c2 2 a2 – c2 . b2 – c2
i.e. if, c2 =  a2 – c2 . b2 – c2
Again, squaring both sides we get
c4 = (a2 – c2) . (b2 – c2) r1 r2
i.e. if, c4 = a2b2 – a2c2 – b2c2 + c4 C1 C2

i.e. if, a2c2 + b2c2 = a2b2


Dividing both sides by a2b2c2 we get

272 | Pioneer Mathematics Grade XI (Approved by CDC Nepal)


a2c2 b2c2 a2b2
i.e. if 2 2 2 + 2 2 2 = 2 2 2
abc abc abc
1 1 1
i.e. if a2 + b2 = c2
Example 10:
Find the equations of tangents to x2 + y2 = 65 from the point (11,3). Also find the angle
between two tangents.
Solution:
Given equation of the circle is x2 + y2 = 65.........(i)
Here, centre of the circle = (0,0) and radius of the circle = 65
The equation of any line through the point (11,3) having slope m is
y –3 = m (x –11)
or, y –3 = mx –11m
or, mx – y + 3 –11m = 0.........(ii)
The line (ii) will be a tangent to the circle (i), if the length of perpendicular from the
centre of the circle to the line is be equal to the radius of the circle.
11m –3
i.e. ± = 65
1 + m2
or, ±(11m –3) = 65 . 1 + m2
or, (11m – 3)2 = (1 + m2).65
or, 121m2 – 66m + 9 = 65 + 65m2
or, 56m2 –66m –56 = 0
or, 28m2 –33m –28 = 0
33 
1089 + 3136 33  4225 33  65
or, m = 56 = 56 = 56
Taking +ve sign and Taking – ve sign
33 + 65 33 –65
m = 56 m = 56
98 32
= 56 = – 56
7 4
=4 = –7
7 7
When m = 4 , then equation of tangent is y – 3 = 4 (x – 11)
or, 4y –12 = 7x –77
or, 7x –4y = 65.............(iii)
4 4
And, When m = – 7 , the equation of tangent is y –3 = –7 (x –11)
or, 7y –21 = –4x + 44
or, 4x + 7y = 65 .......(iv)
7 4
Here, slope of tangent (iii) if (m1) = 4 and slope of tangent (iv) if (m2) = – 7

(Approved by CDC Nepal) Analytical Geometry | 273


7 4
As, m1 × m2 = 4 × – 7 = –1
So, angle between them is 90°.
Example 11:
Determine the value of k so that the length of the tangent from (5,4) to the circle
x2 + y2 +2ky = 0 is 5.
Solution:
Given equation of a circle is x2+y2+2ky = 0
The length of tangent from (5,4) to the circle = 5.
Using the formula, the length of the tangent from (x1,y1) to the circle
x2 + y2 + 2gx + 2fy + c = 0 is x12 + y12 + 2gx1 + 2fy1 + c
The length of tangent from (5,4) to the circle x 2 + y2 + 2ky = 0 is 52+42+2.k.4 .
or, 52 + 42 + 2.k.4 = 5 [∵ By question]
or, 25 + 16 + 8k = 25
or, 8k = –16
 k = –2
Example 12:
Find the locus of point from which two tangents to the circle x 2 + y2 – a2 = 0 are
perpendicular to each other.
Solution:
Given equation of the circle is
x2 + y2 = a2.....(i)
(h,k)
Let, P(h,k) be a point from which the two tangents are drawn a 2
to the circle (i)
The equation of tangents are
y = mx  a 1 + m2
Since these tangents pass through P(h,k) then
k = mh  a 1 + m2
or, (k – mh) =  a 1 + m2
or, (k – mh)2 = a2(1 + m2) (squaring both sides)
or, k2 – 2hkm + m2h2 – a2 – a2m2 = 0
or, (h2 – a2) m2 – 2hkm + (k2 – a2) = 0...........(ii)
Whi is a quadratic equation in m the give two values of m. Say m1 and m2.
k2 – a2
Then, m1m2 = h2 – a2 ( product of roots = c/a in of)
k2 – a2
or, –1 = h2 – a2 [ tangents are perpendicular, m1m2 = –1]
or, –h2 + a2 = k2 – a2
or, h2 + k2 = 2a2
 The required locus of the point P(h,k) is x2 + y2 = 2a2
Remarks: The locus is also a circle concentric with given circle (i) having radius 2a

274 | Pioneer Mathematics Grade XI (Approved by CDC Nepal)


EXERCISE 3 (C )
1. Find the equation of tangents and normal to the following circles at the points
mentioned.
(a) x2 + y2 = 5 at (1,2)
2 2
(b) x + y = 25 at (3,4)
2 2
(c) x + y = 100 at (1,2)
2 2
(d) x + y = 169 at (12,–5)
2 2
(e) x + y – 2x –4y + 3 = 0 at (2,3)
2 2
(f) x + y –3x + 4y –31 = 0 at (–2,3)
2. Find the length of the intercepts made by
(a) the straight line x + y = 3 with the circle x 2 + y2 –2x –3 = 0
(b) the straight line 2x – y = 7 with the circle x2 + y2 –6x –8y + 15 = 0
3. (a) Show that the line 3x + 4y –20 = 0 touches the circle x2 + y2 = 16 and find the point
of contact.
(b) show that the line 3x –4y + 11 = 0 is a tangent to the circle x2 + y2 – 8y + 15 = 0
(c) show that the line 3x –4y = 25 and the circle x2 + y2 = 25 intersect in two coincident
points.
4. Find the equation of the tangent to the circle
(a) x2 + y2 = 25 inclined at an angle 60° to the x–axis.
(b) x2 + y2 = 9 parallel to 3x + 4y = 0
(c) x2 + y2 – 6x + 4y = 12 and parallel to the line 4x + 3y + 5 = 0
(d) x2 + y2 = 5, which are perpendicular to the line x + 2y = 0
(e) x2 + y2 –2x –4y –4 = 0 which are perpendicular to the line 3x – 4y = 1
(f) x2 + y2 = 25 which passes through the point (7,1)
(g) x2 + y2 = 10 at the point whose abscissa is 1.
(h) x2 + y2 – 2ax = 0 at (a(1 + cos), asin)
(i) x2 + y2 – 2x – 4y –20 = 0 which pass through the point (8, 1).
5. Find the value of k if
(a) the line 2x – y + k = 0 may touch the circle x2 + y2 = 5.
(b) the line 4x + 3y + k = 0 may touch the circle x2 + y2 –4x + 10y + 4 = 0
(c) the line 4x –3y + k = 0 is a tangent to the circle x2 + y2 – 8x + 12y + 3 = 0
(d) the line xcos+ysin=k is a tangent to the circle x2+y2 – 2axcos – 2aysin = 0
(e) the line 4x + 3y + k = 0 may touch the circle 2x2 + 2y2 = 5x.
6. Prove that the tangent to the circle x2 + y2 –4x = 0 at the points (1,–1) and (–5,–7) are
parallel.
7. (a) Find the condition that the line lx + my + n = 0 is a tangent to the circle
x2 + y2 = a2.

(Approved by CDC Nepal) Analytical Geometry | 275


(b) Find the condition that the line lx + my + n = 0 may be a tangent to the circle x 2 +
y2 + 2gx + 2fy + c = 0
(c) Find the condition that the line lx + my + n = 0 should be a normal to the circle
x2 + y2 + 2gx + 2fy + c = 0
x y
(d) Find the condition that the line a + b = 1 may touch the circle

(x – a)2 + (y – b)2 = r2,


8. Find the condition for the two circles x2 + y2 = a2 and (x – c)2 + y2 = b2 to touch
(i) externally (ii) internally
9. Find the equation of the circle whose centre is at the point (h,k) and which passes
through the origin and prove that the equation of the tangent at the origin is
hx + ky = 0.
10. Find the equation of tangents, to each circle, through the given external point.
(a) x2 +y2 = 25 from (13,0) (b) x2 + y2 = 10 from (4,–2)
(c) x2 + y2 = 8 from (5,–1) (d) x2 + y2 – 2x + 4y = 0 from (0,1)
11. Find the length of the tangent to the circle
(a) x2 + y2 = 9 from (3,2) (b) 2x2 + 2y2 = 5 from (4,–1)
(c) x2 + y2 –2x –3y –1 = 0 from (2,5) (d) x2 + y2 –4y –15 = 0 from (4,5)
12. Find the value of k so that
(a) the length of the tangent from (5,4) to the circle x 2 + y2 + 2ky = 0 is 1.
(b) the length of the tangent from (4,5) to the circle x 2 + y2 –ky –5 = 0 is 5.
13. Prove that the straight line y = x + a 2 touches the circle x2 + y2 = a2 and find the
point of contact.
14. Find the equation of the line through the point (1,–1) which cuts off a chord of length
4 3 from the circle x2 + y2 –6x –4y –3 = 0

ANSWERS
1. (a) x + 2y = 5, 2x – y = 0
(b) 3x + 4y = 25, 3y – 4x = 0
(c) x + 2y = 100, 2x – y = 0
(d) 12x – 5y = 169, 5x + 12y = 0
(e) x + y –5 = 0, x – y + 1 = 0
(f) 7x – 10y + 44 = 0, 10x + 7y –1 = 0
2. (a) 2 2 (b) 2 5

3. (a) 12  16


5 5
4. (a) y = 3x 10 (b) 3x + 4y + 15 = 10, 3x + 4y –15 = 0
(c) 4x + 3y + 19 = 0, 4x + 3y –31 = 0

276 | Pioneer Mathematics Grade XI (Approved by CDC Nepal)


(d) 2x – y  5 = 0
(e) 4x + 3y + 5 = 0,4x + 3y –25 = 0
(f) 4x – 3y –25 = 0,3x + 4y –25 = 0
(g) x + 3y –10 = 0, x –3y –10 = 0
(h) xcos + ysin = a(1 + cos) ycos = xsin – asin
(i) 4x + 3y –35 = 0, 3x –4y –20 = 0
5. (a)  5 (b) 32 or –18 (c) 1 or –69
5 45
(d) 0 or 2a (e) 4 or – 4

7. (a) a2(l2 + m2) = n2


(b) (g2 + f2 – c) (l2 + m2) = (n – gl – fm)2
1 1 1
(c) n = gl + mf (d) a2 + b2 = r2

8. (i) a+b=c (ii) a – b = c


2 2
9. x + y – 2hx –2ky = 0
10. (a) 12y = 5 (x –13)
(b) 3x + y –10 = 0, x –3y –10 = 0
(c) x + y –4 = 0, 7x –17y –72 = 0
(d) 2x – y + 1 = 0, x + 2y –2 = 0
29
11. (a) 2units (b) 2 units (c) 3 units (d) 6 units

11
12. (a) –5 (b) 5

13.  –a  a 
 2 2
14. 5x – 12y – 17 = 0

(Approved by CDC Nepal) Analytical Geometry | 277


Objectives Questions

Circle the correct Answers


1. The equation x2 + y2 + kx + 6y + 13 = 0 represents a pint circle if k is equal to
a. 2 b. -2 c.  d. -3
2 2
2. One and of a diameter of the circle x +y -3x+5y-4=0 is (2,1) then, the co-ordinates of the other end
is
a. (4,2) b. (3,-6) c. (-2,5) d. (1,-6)
2 2
3. The greatest distance of a point A(10,7) from the circle x +y - 4x - 2y - 20 = 0 is
a. 10 b. 15 c. 5 d. None
2 2
4. The intersect made by the circle x +y - 4x - 2y - 20 = 0 with x-axis is
a. 6 b. 2 6 c. 3 6 d. 4 6
2 2 2
5. For what value of a, the equation x + y + (a - 4) xy + 2x + 2y + a = 0 represents a circle?
a. 1 b. 2 c. 3 d. None of these
6. The lines 2x-3y-5=0 and 3x-4y=7 are the diameters of a circle of area 49 square units, then equatio
of circle is
a. x2 + y2 - 2x + 2y - 47 = 0 b. x2 + y2 + 2x - 2y - 6 =0
c. x2 + y2 - 2x + 2y - 47 = 0 d. x2 + y2 - 2x + 2y - 62 = 0
7. The circle that can be drawn to touch the coordinate axes and the lines 4x+3y=12 cannot lie in the
a. 1st quadrant b. 2nd quadrant c. 3rd quadrant d. 4th quadrant
8. If (h,3) and (3,5) are the extremities of the diameter of the circle with centre (2,k), then the value of
(h,k) is
a. (4,1) b. (1,4) c. (4,4) d. (2,-2)
9. The ara of portion of the circle x2 + y2 - 4y = 0 lying below x-axis is
a. 0 b. 2 c. 3 d. 4
10. The AM of abscissa of points of intersection of the circle x2 + y2 2gx + 2fy + c = 0 with x-axis is
a. g b. -g c. f d. -f

Answers:

1 c 2 d 3 b 4 d 5 d
6 a 7 c 8 b 9 a 10 c

278 | Pioneer Mathematics Grade XI (Approved by CDC Nepal)


3.4

Learning Objective

After the completion of this chapter, students will be able to understand


 find the standard equation of parabola.
 find the equations of tangent and normal to a parabola at given point

(Approved by CDC Nepal) Analytical Geometry | 279


The Greek Scholars of antiquity were the first to study conic sections. With no practical
applications in mind mathematicians pursued the topic solely. Around 225 B.C.E. the
scholars 'Apollonius' of Perga wrote a series of books and titled conics in which he thoroughly
investigated these curves and introduced the name PARABOLA, ELLIPSE and HYPERBOLA.
Almost 2,000 years later, scientists began finding applications of conic sections to problems in
the real world. In 1604, 'Galileo Galilei' discovered that objects thrown in the air follow
parabolic paths.
In 1609, 'Johannes Kepler' (an astronomer) discovered that the orbit of Mars is an elliptical
and he concluded that all planetary bodies have elliptical orbits. 60 years later, Sir Isaac
Newton was able to prove this fact by using his newly developed law of gravitation. In the
same time scientists have discovered that the path of an alpha particle in the electrical field of
an atomic nucleus is a hyperbola.
Eccentricity
The locus of a point which moves in a plane in such a way that the ratio of its distance from a
fixed point to its distance from a fixed straight line is constant is called the conics and the
fixed ratio is called the eccentricity (e).
The fixed point is its focus, the fixed straight line is it directrix and the straight line passing through line
passing though the focus and perpendicular to the directrix is called the axis of the conic. The distance
of any point on a conic from the focus is called a focal distance or focal radius of the point. A point of
intersection of the conic and the axis is called the vertex.
A conic is said to be:
 A circle if eccentricity (e) = 0
 A parabola if eccentricity (e) = 1
 A ellipse if eccentricity (e) < 1
 A hyperbola if e > 1.

Fig: Conic sections

Parabola
A parabola is the plane curve consisting of all points that are equally distant from a given
point and a given fixed line. The fixed point is its focus and the fixed line is its directrix.
A parabola also arises as the curve produced by the intersection of a plane through a right
circular can held parallel to the slant side of the cone.

280 | Pioneer Mathematics Grade XI (Approved by CDC Nepal)


A parabola can also be defined in terms of eccentricity and is defined as a conic whose
eccentricity is unity is called parabola.
There are wide applications of parabolic curves in science and engineering. For examples,
satellite dishes and reflecting telescope use dishes with parabolic cross sections so as to focus
parallel rays of light to a fixed point. Conversely searching light reflectors and automobiles
head light reflectors reflect all the rays from a bulb positioned at the focus, parallel to the axis
of the parabola. Radder signals and sound waves also use parabolic shape of many antennas.

Fig: Applications of parabola

Outgoing light parallel to axis


Parabolic light
reflector

Filament (point soure) at focus

HEADLAMP RADIO TELESCOPE


Fig: Applications of parabola

Equation of a Parabola Y
P(x, y)
(A) Equation of a parabola in a standard form M
(–a, y)
Any parabola whose vertex is at origin O and its axis
is either x-axis or y-axis are the parabola in the
standard position. O X
X'
Z a a S(a, 0)
Let S be a focus and ZM be the directrix of the A
(–a, 0)
parabola. Draw a line SZ perpendicular to ZM. Let A
be the middle point of SZ so that
SA = AZ = a (say) and the co-ordinates of Z, A and
Y'
S are (–a, 0), (0, 0) and (a, 0) respectively. Here A is Fig: Equation of parabola in a standard form
the vertex and ZAS is the axis of the parabola which
is x-axis itself.
Let P(x, y) be any point on the parabola.
Join PS and draw PM perpendicular to ZM. So that the co-ordinate of M be (–a, y).
Then by definition of a parabola,
PS
eccentricity (e) = PM = 1

(Approved by CDC Nepal) Analytical Geometry | 281


or, PS = PM
or, PS2 = PM2
or, (x – a)2 + (y – 0)2 = [x – (–a)2 + (y – y)2]
or, x2 – 2ax + a2 + y2 = (x + a)2
or, x2 – 2ax + a2 + y2 = x2 + 2ax + a2
or, y2 = 4ax
As the point P on the parabola is arbitrary, the relation is true for every point P that lie on a
parabola. Thus the equation of a parabola is y2 = 4ax.
Remarks:
(a) The points (at2, 2at) and (at2, –2at) always lie on the parabola y 2 = 4ax. Where –<t<.
(b) Any two points on the curve are (at12, 2at1) and (at22, 2at2).
Example 1:
Find the equation of the parabolic satellite dish whose vertex is at (0, 0) and satellite dish
focuses parallel rays of light to a fixed point (5, 0).
Solution:
We have; vertex = A(0, 0) = origin
the fixed point = focus = S(a, 0) = S(5, 0)
So, the equation of parabolic satellite dish is
y2 = 4ax
or, y2 = 4.5x
or, y2 = 20 x
Types of parabola in a standard form
There are two types of parabola in a standard form and they are:
(a) Horizontal parabola
(b) Vertical parabola
(a) Horizontal parabola
The standard parabola which is opened to the right or left is called the horizontal
parabola and their equations are y2 = 4ax and y2 = – 4ax respectively.
For example: y2 =16x = 4.4 x is right opened parabola and
y2 = –16x = 4(–4)x is left opened parabola
In which a = 4 > 0 and a = –4 < 0 respectively.
Directrix Y Y Directrix
y2 = 4ax y2 = –4ax

(focus) (focus) O, A
O (axis)
(axis)
X' A S(a, 0) X X' S(–a, 0) (0, 0) X

x = –a Y' Y'
(i) Right opened (ii) Left opened

Fig: Horizontal parabola

282 | Pioneer Mathematics Grade XI (Approved by CDC Nepal)


(b) Vertical parabola
The standard parabola which is opened upward or downward is the vertical parabola and
their equations are x2 = 4ay and x2 = –4ay respectively.
For example: x2 = 24y = 4.6 y is upward opened parabola and
x2 = –8y = 4(–2)y is downward opened parabola
Y x2= 4ay Y

axis
focus (0, a) Directrix y=a
A O(0, 0)
vertex
vertex
A O(0, 0)
Directrix
focus (0, –a)
y = –a
axis

Y' Y' x2 = –4ay


(i) Vertically upward (ii) Vertically upward
Fig: Vertical parabola

Terms related in parabola


Focal distance: The distance of any point on a parabola from the focus is called the focal
distance or focal radius of the point.
Focal length: The distance of the focus from the vertex is called the focal length of a parabola
Focal chord: Any chord of a parabola passing through the focus is called the focal chord of a
parabola.
Latus rectum: The focal chord which is perpendicular to the axis of the parabola is called the
latus rectum or focal width of the parabola. Y
If y2 = 4ax be the equation of a parabola and P be any
point on the parabola, A be the vertex and S be the P
focus. Then in the figure. y2 = –4ax
M
PS is focal distance of a point P.
AS is focal length O
X' A S X
MM' is the focal chord
LL' is the latus rectum of a parabola.
The co-ordinate of end points of a latus rectum are L'
M'
L(a, 2a) and L'(a, –2a). (a, –2a)
Shifting of a Parabola Y'
(a) Reflection: If a > 0, then Fig: Various components in parabola
2 2
(i) If y = 4ax be a parabola then its reflection about the y-axis is y = –4ax
(ii) If x2 = 4ay be a parabola then its reflection about the x-axis is x2 = –4ay

(Approved by CDC Nepal) Analytical Geometry | 283


Y Y
y2 = 4ax y2 = –4ax

O O, A
X' A S(a, 0) X X' S(a, 0) (0, 0) X

Y' Y'
Fig: Reflection about y-axis
Y x2= 4ay
Y

S(0, a)

X' X X' X

S(0,–a)

Y'
Y' x2 = –4ax

Fig: Reflection about x-axis


(b) Horizontal shifting: If h > 0, then
(i) If y2 = 4ax be a parabola then y2 = 4a(x – h) is the right horizontal shifting of a
parabola y2 = 4ax by h units.
(ii) If y2 = 4ax be a parabola then y2 = 4a(x + h) is the left horizontal shifting of a
parabola y2 = 4ax by h units.
Y Y Y
y 2 = 4ax y 2 = 4a( x –h) y 2 = 4a(x + h)

O O h a
X' A S ( a, 0) X X' A S(( a+ h) , 0) X X' A h O S ( a–h) ,0) X

Y' Y' Y'

F i g: H or i zontal shi fti ng

(c) Vertical shifting: If k > 0, then


(i) If x2 = 4ay be a parabola then x2 = 4a(y – k) is the vertically up shifting of a
parabola by k units.

284 | Pioneer Mathematics Grade XI (Approved by CDC Nepal)


(ii) If x2 = 4ay be a parabola then x2 = 4a(y + k) is the vertically down shifting of a
parabola by k units.
Y x2 = 4a(y–k)
Y x2 = 4ay Y
S(0,a+k)
x2 = 4a(y+k)
S(0,a)
x A(0,k) S(0,a–k))
X' O A X X' O X X' O X
A(0,–k)
Y' Y'
Y'
Fig: Vertical shifting
(d) Both horizontal and vertical shifting: for h, k > 0 then
(i) If y2 = 4ax be a parabola then (y – k)2 = 4a (x – h) is the parabola obtained by
shifting the parabola right by h units and up by k units.
(ii) If y2 = 4ax be a parabola then (y + k)2 = 4a (x + h) is the parabola obtained by
shifting the parabola left by h units and down by k units.
(y–k)2 = 4a(x –h)
Y Y
y2 = 4ax

(h,k)
A S (a+ h , k)
k
O O
X' A S (a, 0) X X' h X

Y' Y'
Y (x –k)2 = 4a(x –h)
Y x 2 = 4ay
S (h+ a,k)
S (0,a)

k A (h,k)
X' O A X
X' O h X

Y' Y'
F i g: B oth hor i zontal and ver ti cal shi fti ng

(Approved by CDC Nepal) Analytical Geometry | 285


(B) The equation of a parabola with its axis parallel to the x-axis and vertex at any point
Let A(h, k) be the vertex of a parabola whose axis is parallel to the x-axis and its focus is
at a distance 'a' right from the vertex so that S(h + a, k) is the focus and ML, a line
parallel to y-axis, is the directrix. Y
Let P(x, y) be any point on the parabola. P(x, y)
Draw PM perpendicular to the directrix ML
T M
so that RA = h, AS = a, NA = AS = a
and TM = RN = RA – NA = h –a R N
A a
Thus, the equation of directrix is x = h – a a-h S(a+h,k)
and the co-ordinate of M is (h – a, k) k
By definition of a parabola, we have, X'
O
X
h
PS
e = PM = 1
Y' L
or, PS = PM
Fig: Parabola with its axis paralle
or, PS2 = PM2 to x-axis and vertex at (h, k)
By using distance formula, we get,
[x – (a + h)]2 + (y – k)2 = [x – (h – a)]2 + (y – y)2
or, (x – a – h)2 + (y – k)2 = (x – h + a)2
or, (y – k)2 = (x – h + a)2 – (x – a – h)2
Using a2 – b2 = (a + b) (a – b) on right side, we get,
(y – k)2 = [(x – h + a) + (x – a – h)] [(x – h + a) – (x – a – h)]
or, (y – k)2 = [2(x – h)] 2a
or, (y – k)2 = 4a (x – h) is the required equation of the parabola whose vertex is at (h, k)
and its axis is parallel to x-axis.
(C) The equation of a parabola with its axis parallel Y (x–h)2 = 4a(y–k)
to y-axis and vertex at any point:
By the similar process to that of equation of a a
S(h,a+k)
parabola with its axis parallel to x-axis and vertex
A(h,k)
at any point (h, k), we can find the equation of a k a
parabola with its axis parallel to y-axis and vertex
at (h, k) is, X' O h X
2
(x – h) = 4a(y – k) Y'
Fig: Parabola with its axis paralle
to y-axis and vertex at (h, k)
Example 2:
(a) Find the equation of the parabola in which vertex is at (2, 3) and searching light reflector
reflect all the rays from (4, 3) parallel to the axis of parabola.
(b) Vertex is at (1, 3) and reflecting telescope dish focus parallel rays of light to a fixed point
(1, 5)
Solution:
(a) We have, vertex = A(2, 3) = A(h, k) so that h = 2, k = 3

286 | Pioneer Mathematics Grade XI (Approved by CDC Nepal)


Focus = S(4, 3) = S(h + a, k) so that h + a = 4  a = 4 – 2 = 2
As, y co-ordinate in both vertex and focus is same, the axis of the parabola is parallel to
x-axis and its equation is
(y – k)2 = 4a(x – h)
or, (y – 3)2 = 4.2 (x – 2)
or, (y – 3)2 = 8 (x – 2)
(b) We have, vertex = A (1, 3)= A(h, k) so that h = 1, k = 3.
Focus = S(1, 5) = S(h, k + a) so that k + a = 5  a = 5 – 3 = 2
As, x co-ordinate in both vertex and focus is same, the axis of the parabola is parallel to
y-axis, and its equation is,
(x – h)2 = 4a(y – k)
or, (x – 1)2 = 4.2 (y – 3)
or, (x – 1)2 = 8 (y – 3)

Results in Parabolic Equations


Equation of parabola for a > 0
2 2
y = 4ax (y–k) = 4a (x–h) x2 = 4ay (x–h)2= 4a(y–k)
Vertex (0, 0) (h, k) (0, 0) (h, k)
Focus (a, 0) (h + a, k) (0, a) (h, k + a)
Equation of directrix x = –a x=h–a y = –a y=k–a
Axis of parabola x-axis y=k y-axis x=h
Length of latus rectum 4a 4a 4a 4a
End points of latus (a,2a), (h+a,k+2a) and (2a, a), (–2a, a) (h + 2a, k + a)
rectum (a,–2a) (h + a, k – 2a) and (h–2a,k+a)
Opened to right right up Up
Symmetricity x-axis y=k y–axis x=h
(symmetric about)
Parametric equation x =at2 x = h + at2 x = 2at x = h + 2at
for –<t< y = 2at y = k+2at y = at2 y = k + at2
Example 4:
Find the co-ordinates of the focus, the vertex, the equation of directrix, the length of a latus
rectum and the axis of the following parabola.
(a) y2 = –8x
(b) x2 = 6y
(c) (y –5)2 = 2(x + 2)
(d) x2 + 2x + 8y – 15 = 0
Solution:
(a) We have, the given equation of a parabola is

(Approved by CDC Nepal) Analytical Geometry | 287


y2 = –8x ………. (i)
Comparing (i) with y2 = 4ax, we get,
4a = –8  a = –2
Whose vertex is at (0, 0)
focus = (a, 0) = (–2, 0)
Also, the equation of directrix is
x = –a
or, x = –(–2)
or, x = 2
Again, the length of a latus rectum = 4|a| = 4|–2| = 8 units.
Finally, As, the parabola y2 = 4ax has its axis as x-axis, so its axis y = 0
(b) We have the equation of a parabola is Y
x2 = 6y ………….. (i)
Comparing (i) with x2 = 4ay, we get,
3
4a = 6  a = 2
A ( 0, 0)
Whose vertex is at (0, 0) X' X
S ( –2, 0)
3
focus = (0, a) = 0 2
 
Also, the equation of directrix is
y = –a y 2 = –8x Y' x=2
3
or, y = – 2 F i g: E xampl e 4
or, 2y + 3 = 0
3
Again, the length of a latus rectum = 4|a| = 4| 2 | = 6 units
Finally, as, the parabola x2 = 4ay has its axis y-axis, so its axis is x = 0
(c) We have, the given equation of a parabola is
(y – 5)2 = 2(x + 2)
Which can be written in the form of
1
(y – 5)2 = 4 . 2 (x – (–2)) ………. (i)
Comparing (i) with (y – k)2 = 4a(x – h), we get,
1
h = –2, k = 5 and a = 2
1 –3
Then, focus = (h + a, k) = –2 + 2 5 =  2  5
   
vertex = (h, k) = (–2, 5)
Again, the equation of directrix is,
x = h –a
1
or, x = –2 –2
or, 2x = –5
or, 2x + 5 = 0

288 | Pioneer Mathematics Grade XI (Approved by CDC Nepal)


1
Again, the length of a latus rectum = 4|a| = 4| 2 | = 2 units
As the equation of a parabola, has its axis parallel to x-axis, by k = 5 unit, so the axis of
the parabola is y = 5.
Finally,
(d) We have, the given equation of a parabola is
x2 + 2x + 8y – 15 = 0
Which can be written in the form as,
(x2 + 2x + 12) – 1 = –8y + 15
or, (x + 1)2 = –8y + 16
or, (x – (–1))2 = –8(y – 2)
or, (x – (–1))2 = 4.(–2) (y – 2) …………… (i)
Comparing (i) with (x – h)2 = 4a (y – k), we get,
h = –1, k = 2, and a = –2
Then, focus = (h, k + a) = (–1, 2 + (–2)) = (–1, 0)
vertex = (h, k) = (–1, 2)
Again, the equation of directrix of the parabola
y=k–a
or, y = 2 – (–2)
or, y = 4
Now, the length of a latus rectum = 4|a| = 4|–2| = 8 units
Finally, the equation of a parabola has its axis parallel to y-axis by h = –1 unit. So the
parabola has its axis x = –1.
Example 5:
At what points of the parabola y2 = 9x is the ordinate three times that of its abscissa?
Solution:
We have the given parabola is, y2 = 9x ……….. (i)
Let, the required point on the parabola be (x, y), then by question
ordinate = 3 times abscissa
i.e. y = 3x
or, y2 = 9x2 …………… (ii)
From (i) and (ii), we get,
9x = 9x2
or, x2 – x = 0
or, x(x – 1) = 0
This gives x = 0 and x = 1
From (i), when x = 0, y = 0
x = 1, y2 = 9.1
 y=+3
Thus, the required points are (0, 0), (1, 3) and (1, –3)

(Approved by CDC Nepal) Analytical Geometry | 289


Example 6:
Show that the lines joining the ends of the latus rectum of the parabola y 2 = 4ax to the
point of intersection of the directrix and the axis are at right angles.
Solution:
directrix Y
We have, the given equation of a parabola,
y2 = 4ax whose vertex is at A(0, 0), focus is
at S(a, 0) and its directrix is x = –a so that
AS = AD = a.
O
Thus, the co-ordinate of point of X' D A S(a, 0) X
intersection of directrix and axis of parabola
is D(–a, 0) and the co-ordinate of ends of
latus rectum are L(a, 2a) and L' (a, –2a).
x=2 Y'
Now,
Fig: Example 6
Let (–a, 0) = D (x1, y1) and l (a, 2a) =l (x2, y2).
y –y 2a – 0 2a
Slope of LD is = x2 – x1 = a – (a) = 2a = 1
2 1

For the slope of L'D


Let D (–a, 0) = D (x1, y1) and l1 (a, – 2a) = l1 (x2, y1)
–2a – 0 –2a
 the slope of L'D is = a – (–a) = 2a = –1

Here, the slope of LD × slope of L'D = 1 × (–1) = –1


This shows that, the lines joining the ends of latus rectum of the parabola y 2 = 4ax to the
point of intersection of the directrix and the axis are at right angles.
Example 7:
Find the locus of the mid-points of the chords of the parabola y2 = 4ax passing through the
vertex, prove that it is parabola. Also, find its focus and latus rectum.
Solution:
Let P(x, y) be the any point on the parabola y 2 = 4ax and
Y
M(h, k) be the mid-point of the chord AP so that, P y 2= 4ax
0+x 0+y x y
(h, k) =  2  2  = 2  2
    y 2= 2ax
M ( h, k )
x y
Which gives h = 2 and k = 2 O
X' A S ( a, 0) X
 x = 2h and y = 2k
Substituting the value of x and y in (i)
(2k)2 = 4a . 2h
or, k2 = 2ah Y'
Thus, the locus of the mid-point of the chord is y2 = 2ax F i g: E xampl e 7
a a
Which can be written in the form y 2 = 4 2 x represents a parabola with focus at 2  0
   
a
and latus rectum = 4 . 2 = 2a.

290 | Pioneer Mathematics Grade XI (Approved by CDC Nepal)


Example 8:
Find the parametric equation of the following parabola.
(a) y2 = 6x
(b) x2 = –8y
Solution:
3
(a) We have, the given parabola is, y2 = 6x which can be written in the form y 2 = 42 x
3
comparing (i) with y2 = 4ax, we get, a = 2 ………….. (i)

Since, the point (at2, 2at) always lies on the parabola y2 = 4ax,
the equation of parabola in the parametric form are;
3 3
x = at2 = 2 t2 and y = 2at = 2.2 t = 3t.

3
x = 2 t2 and y = 3t are the parametric equation.

(b) We have, the given parabola is x2 = –8y


Which can be written in the form x2 = 4(–2)y ………….. (ii)
2
Comparing (i) with x = 4ay we get, a = –2
Since, the point (2at, at2) always lies on the parabola x2 = 4ay, the equation of parabola
in parametric form are;
x = 2at = 2.(–2)t = –4t and y = at2 = –2t2
or, x = –4t and y = –2t2 are the parametric equation.

EXERCISE 3. 4 (A)
1. Find the equations of the parabola in the following cases:
(a) Vertex at (0, 0) and focus at (3, 0).
–3
(b) Vertex at (0, 0) and focus at  2 0
 
(c) Vertex at (0, 0) and focus at (0, 4).
(d) Vertex at (3, 2) and focus at (6, 2).
(e) Vertex at (5, 3) and focus at (5, 6).
Also, find the focal length and find where the curve is opened.
2. Find the equation of the parabola in the following case.
(a) Focus at (4, 0) and its directrix is x = –4
(b) Vertex at the origin, passing through the point (5, 2) and symmetric about
y-axis.

(Approved by CDC Nepal) Analytical Geometry | 291


(c) The length of a latus rectum is 16, its axis parallel to the x-axis and passing through
(3, 2) and (3, –2)
3. Find the co-ordinates of the (i) vertex, (ii) focus, (iii) the axis of a parabola, (iv) the
equation of a directrix, (v) the length of a latus rectum and (vi) the ends of a latus rectum
for the following equations of a parabola.
(a) 3y2 = 8x
(b) x2 = –8y
(c) (y – 2)2 = 2(x + 3)
(d) x2 – 4x – 3y + 13 = 0
4. Find the focal distance of a point P for the following cases:
(a) If P(–4, 2) and parabola is y2 = 8x
(b) If P(4, 1) and parabola is x2 = 16y
5. At what points on the parabola y2 = 36x is the ordinate two times that of its abscissa.
6. Find the area of the triangle formed by the lines joining the vertex of the parabola
x2 = 12y to the ends of its latus rectum.
7. Find the locus of the mid-points of the chords of a parabola y2 = 4ax, drawn from focus.
Prove that the locus is a parabola.
8. A double ordinate of the curve y2 = 4ax is of length 8a. Prove that the lines joining the
vertex to its ends of the latus rectum are at right angles.
9. Find the parametric equation of the following parabola.
(a) y2 = 10x
(b) x2 = – 16y
(c) (y – 2)2 = 2(x + 3)

ANSWERS
3
1. (a) y2 = 12x, 3, right opened (b) y2 = –6x, 2 , left opened
(c) x2 = 16y, 4, opened upwards (d) (y – 2)2 = 12 (x – 3), 3, left opened
(e) (x – 5)2 = 12(y – 3), 3, opened upward
2. (a) y2 = 16x (b) 2x2 = 25y (c) 4y2 = 4(4x – 13)
2
3. (a) (i) (0, 0), (ii) 3  0 , (iii) y = 0, (iv) 3x + 2 = 0,
 
8 2 4 2 –4
(iv ) 3 unit (vi) 3 3 and 3 3 
   
(b) (i) (0, 0), (ii) (0, –2), (iii) x = 0, (iv) y = 2 (v) 8 units (vi) (–4, 2) and (4, –2)
–5
(c) (i) (–3, 2), (ii)  2  2 , (iii) y = 2, (iv) 2x + 7 = 0, (v) 2 unit,
 
–5 –5
(vi)  2  3 and  2  1
   
15
(d) (i) (2, 3) (ii) 2 4  (iii) x = 2, (iv) 4y – 9 = 0, (v) 3 unit,
 
(vi) 7 15 and 1 15
2 4  2 4 

292 | Pioneer Mathematics Grade XI (Approved by CDC Nepal)


4. (a) 2 10 unit (b) 5 unit
5. (0, 0) and (9, 18) 6. 18 sq. unit
5 1
9. (a) x = 2 t2, y = 5t (b) x = –8t, y = –4t2 (c) x = –3 + 2 t2 , y = 2 + t

Objectives Questions
Circle the correct Answers:
1. The equation x2 + 4xy + y2 + x+3y+2 = 0 represents a parabola if  is
a. 0 b. -4 c. 4 d. None
2
2. The eccentricity of the parabola x =4x -4y + 4 = 0 is
1 1
a. e>1 b. e= 4 c. e = 2 d. e = 1
3. If the equation of focal chord of the parabola y2 4x is 2x+3y-4=0, then the length of latus
rectum is
a. 2 b. 4 c. 6 d. 8
2
4. If a focal chord of the parabola y = ax is 2x-y-8 =0, then the equation of directrix is
a. x-4 = 0 b. x + 4 = 0 c. y-4 = 0 d. y+4 = 0
2 2
5. The parabola x = 4y and y = 4x intersect
a. in a unique point b. On the line y = x
c. On the lien y = -x d. None
6. Vertex of the parabola y2+2y+x=0 lies in the quadrant
a. first b. second c. third d. fourth
7. The axis of parabola 9y2-16x-12y-57 = 0 is
a. 3y = 2 b. 16x+61=0 c. y=0 d. 6y=5
8. If the parabola y2=4ax passes through (3,2) then length of latus ractum is
3 4 9
a. b. c. d. 4
2 3 2
9. If the line x-1 = 0 is the directrix of the parabola y2 - kx + 8 = 0 then one of the values of k
is
1 1
a. 8 b. 8 c. 4 d. 4
10. If the vertex of the parabola y = x2-16x+k lies on x-axis, then value of k is
a. 8 b. 16 c. -64 d. 64
Answers:

1 c 2 d 3 d 4 b 5 b
6 d 7 a 8 b 9 c 10 d

(Approved by CDC Nepal) Analytical Geometry | 293


Tangents and Normal

A point and a parabola


Let P(x1, y1) be any point and y2 = 4ax be a parabola. Draw a line PR perpendicular to
x-axis so that OR = x1, RQ = 4ax1 and RP = y1. Then, P(x1,y1)
(i) Point P lies outside the parabola Q
if RP > RQ
i.e. if PR2 > RQ2 X'
O R
X
2
i.e. if y1 > 4ax1
(ii) Point P lies on the parabola P(x1,y1)
if PR = RQ Q
i.e. if RP2 = RQ2 X' X
O R
i.e. if y12 = 4ax1
(iii) Point P lies inside the parabola Q
if RP < RQ P(x1,y1)
i.e. i.f RP2 < RQ2 X' X
2
O R
i.e. if y1 < 4ax1
Thus, the point P(x1, y1) lies outside or on or inside the parabola y2 = 4ax according as
y12 > 4ax1, or y12 = 4ax1 or y12 < 4ax1 respectively.
Remarks:
(a) The point P(x1, y1) lies outside or on or inside the parabola x 2 = 4ay according as
x12 > 4ay1 or x12 = 4ay1 and x12 < 4ay1 respectively.
(b) The point P(x1, y1) lies outside or on or inside the parabola (y – k)2 = 4a(x – h) according
as (y1 – k)2 < 4a (x1 – h), (y1 – k)2 = 4a(x1 – h) and (y1 – k)2 < 4a(x1 – h) respectively.
(c) The point P(x1, y1) lies outside or on or inside the parabola (x – h)2 = 4a(y – k) according
as (x1 – h)2 > 4a(y1 – k), (x1 – h)2 = 4a(y1 – k) and (x1 – h)2 < 4a(y1 – k) respectively.
Example 1:
Test whether the points (–5, 2), (1, 2 2 ) and (2, 1) lies inside or on or outise the parabola
y2 = 8x
Solution:
Here, the given parabola is y2 = 8x
For the point, (x1, y1) = (–5, 2)
Now, y12 = (–5)2 = 25 and 4ax1 = 8.2 = 16
As, 25 > 16 i.e. y12 > 4ax1, the point (–5, 2) lies outside the parabola
For the point, (x1, y1) = (1, 2 2 )
Now, y12 = (2 2)2 = 8 and 4ax1 = 8.1 = 8
as, y12 = 4ax1 = 8, the point (1, 2 2) lies on the parabola.
For the point, (x1, y1)= (2, 1)
Now, y12 = 12 = 1 and 4ax1 = 8.2 = 16
As 1 < 16 i.e. y12 < 4ax1, the point (2, 1) lies inside the parabola.

294 | Pioneer Mathematics Grade XI (Approved by CDC Nepal)


A line and a Parabola
Let the given line y = mx + c …..………. (i)
the given parabola y2 = 4ax ………….. (ii)
Solving equation (i) and (ii),
(mx + c)2 = 4ax
or, m2x2 + 2cmx + c2 = 4ax
or, m2x2 + (2cm – 4a) x + c2 = 0 ………….. (iii)
Which is in the form Ax2 + Bx + C = 0.
Where A = m2, B = 2cm – 4a and C = c2
Now, B2 – 4AC = (2cm – 4a)2 – 4.m2c2
= 4c2m2 – 2.2cm . 4a + 16a2 – 4m2c2
or, B2– 4AC = 16a(a – cm)
The following three cases crises.
(i) The roots of quadratic equation (iii) are imaginary
if B2 – 4AC < 0
i.e. if 16a (a – mc) < 0
i.e. if a – mc < 0
i.e. if a < cm
a
i.e. if c > m
a
So the line y = mx+c does not intersect the parabola y2 = 4ax if C> m
(ii) The roots of a quadratic equation (iii) are real and distinct
if B2– 4AC > 0
i.e. if (a – mc) > 0
i.e. if a – cm > 0
i.e. if a > cm
a
i.e. if c < m
a
So the line y = mx+c intersect the parabola y2 = 4ax out two different points if c < m
(iii) The equation (iii) has
Two real and equal roots
if B2 – 4AC = 0
i.e. if 16a (a – cm) = 0
i.e. if a – cm = 0
a a
i.e. if c = m so, y = mx + c meet the parabola of c = m
Remarks: the line y = mx + c will
a
(i) Not intersect the parabola y2 = 4ax if c > m
a
(ii) Intersect the parabola y2 = 4ax at two different points if c < m

(Approved by CDC Nepal) Analytical Geometry | 295


a
(iii) Meet the parabola at the coincident point if c = m and the point of contact is

(ma  2am)
2

a
i.e. the line y = mx + m is always be tangent to the parabola y2 = 4ax
Example 2:
Does the following lines intersect the parabola y2 = 16x? If intersect, find the points.
(a) y = 3x + 2, (b) y = x + 4 and (c) y = 2x + 1
Solution:
We have, the given parabola is y2 = 16x
Which can be written in the form y2 = 4.4x …………. (i)
Comparing (i) with y2 = 4ax, we get, a = 4
(a) Comparing the line y = 3x + 2 with y = mx + c, we get, m = 3 and c = 2
a 4
Here, m = 3 and c = 2.
4 a a
As 3 < 2 gives m < c i.e. c > m
The line y = 3x + 2 does not intersect the parabola y 2 = 16x.
(b) Comparing the line y = x + 4 (ii) with y = mx + c. We get, m = 1 and c = 4
a 4
Here, m = 1 = 4 and c = 4
a
As c = m = 4, the line y = x + 4 on meet the parabola y 2 = 16x at the coincident point.
For the point of contact, solving (i) and (ii)
(x + 4)2 = 16x
or, x2 + 8x + 16 = 16x
or, x2 – 8x + 16 = 0
or, (x – 4)2 = 0
 x = 4, 4
For x = 4, y = 4 + 4 = 8
Thus, the point of contact is (4, 8)
(c) Comparing the line y = 2x + 1 ……… (iii) with y = mx + c. We get, m = 2 and c = 1
a 4
Here, m = 2 = 2 an c = 1
a
As c < m , the line y = 2x + 1 intersect the parabola y 2 = 16x at two different points.
For the point of contact, solving (i) and (iii)
(2x + 1)2 = 16x
or, 4x2 + 4x + 1 = 16x
or, 4x2 – 12x +1 = 0
–(–12) + (–12)2 – 4.4.1 12 + = 128 12+ 8 2
 x = 2.4 = 8 = 8

 x =
 3 + 2 2 
 2 
2 (3 + 2 2)
and y = 2 + 1 = (4 + 2 2)

296 | Pioneer Mathematics Grade XI (Approved by CDC Nepal)


Thus, the point of contact are
3 + 2 2 4 + 2 2 and 3 – 2 2  4 – 2 2
 2   2 
Equation of the tangent and normal to the parabola y 2 = 4ax at a point (x1, y1) lies on a
parabola Y
tangent
For the equation of tangent y2=4ax
2
We have, the given parabola is y = 4ax …. (i)
The given point on the parabola is (x1, y1).
Since, the point (x1, y1) lies on y2 = 4ax or, y12 = 4ax1… (ii)
A, O S(a, 0)
On differentiating (i) w. r. t. 'x', we get, X' X
dy
2y dx = 4a
dy 2a
or, dx = y Y'
Fig: The tangent and normal to the parabola
dy
Since, dx at (x1, y1) gives the slope of the tangent to the curve y = f(x) at that point.
dy 2a
The slope of the tangent to the parabola y 2 = 4ax at (x1, y1) is, m = dx at (x1, y1) = y .
1
Thus, the equation of tangent at (x1, y1) is,
y – y1 = m(x – x1)
2a
or, y – y1 = y (x – x1)
1
or, yy1 – y12 = 2ax – 2ax1
or, yy1 – 4ax1 = 2ax – 2ax1 [ y12 = 4ax1]
or, yy1 = 2a (x + x1)
Thus the equation of tangent at (x1, y1) is yy1 = 2a(x + x1)
For the equation of normal
2a
We have, the slope of the tangent at (x 1, y1) is y .
1
–y
Since, the normal is perpendicular to the tangent, slope of the normal at (x 1, y1) is m = 2a1 .
–y
Thus, the equation of normal at (x1, y1) is y – y1 = 2a1 (x – x1) ………. (iv)
Equation of normal in slope (m) form :
–y
As, we have m = 2a1  y1 = –2am
2
From (ii), (–2am) = 4ax1  x1 = am2
Substituting the value of x1 and y1 in (iv), we get,
y – (–2am) = m(x – am2)
or, y + 2am = mx – am3
or, y = mx – 2am – am3 is the equation of normal in m-form or in the slope form.
Example 3:
Find the equation of tangent and normal to the parabola y2 = 16x at (1, –4).
Solution:
We have the given parabola is y2 = 16x ……… (i) and
The point is (x1, y1) = (1, –4).
On differentiating (i) w.r.t. 'x' we get,
dy
2y dx = 16

(Approved by CDC Nepal) Analytical Geometry | 297


dy 8
or, dx = y
For the equation of tangent
8
At (1, –4), slope of the tangent (m) = –4 = –2
Thus, the equation of tangent is,
y – y1 = m(x – x1)
or, y – (–4) = 2 (x – 1)
or, y + 4 = –2x + 2
or, 2x + y + 2 = 0
For the equation of normal
1
Since, the slope of the tangent at (1, –4) is –2, the slope of the normal is m = 2
Thus, the equation of normal at (1, –4) is
y – y1 = m(x – x1)
1
or, y – (–4) = 2 (x – 1)
or, 2(y + 4) = x – 1
or, x – 2y – 9 = 0
Alternative method
1
We have, the slope of the normal m = 2
The equation of normal at (–4, 1) is,
y = mx – 2am – am3
1 1 1 3
or, y = 2 x – 2.4.2 – 4.2
 
x 1
or, y = 2 – 4 – 2
or, 2y = x – 8 – 1
or, x – 2y – 9 = 0
Example 4:
Find the equation of tangent and normal to the parabola y2 = 4ax in parametric form.
Solution:
We know the equation of a parabola in parametric form is, x = at2 and y = 2at
Therefore, the equation of tangent at (x 1, y1) = (at2, 2at) is
yy1 = 2a (x + x1)
or, y . 2at = 2a (x + at2)
1
or, x – yt + at2 = 0 whose slope is m1 = t and
The equation of normal, whose slope m = –t is,
y – y1 = m(x – x1)
or, y – 2at = –t(x – at3)
or, y + tx – 2at – at3 = 0
Example 5:
Prove that the line lx + my + n = 0 will be
(a) tangent to the parabola y2 = 4ax if ln = am2.
(b) normal to the parabola y2 = 4ax if al (2m2 + l2) + m2n = 0
Solution:
(a) We have the given parabola is y2 = 4ax
and the line is lx + my + n = 0

298 | Pioneer Mathematics Grade XI (Approved by CDC Nepal)


or, my = –lx – n
l n
or, y = –m x – m ……….. (i)
Comparing the line (i) with y = mx + c, we get,
l n
m = – m and c = – m
Since, the line y = mx + c will be tangent to the parabola y 2 = 4ax
a
if c=m
n a
i.e. if – m = –l
m
n am
i.e. if m = l
i.e. if ln = am2 proved
The equation of normal to the parabola is
y = m1x – 2am1 – am13 …………. (ii) where m1 is the slope of the normal
The line (i) and (ii) both will be normal if they are identical.
1 –l/m n/m
i.e. if 1 = m = 2am + am 3
1 1 1
Taking first two ratios, we get,
l
m1 = – m
Taking first and last ratios, we get,
1 n/m
1 = 2am1 + am13
n
or, 2am1 + am13 = m
l
Substituting the value of m1 = – m , we get,
n
2a – m + a – m = m
l l 3
   
[2alm2 + l2] n
or, – m3 =m
or, –al (2m2 + l2) = m2n
or, al (2m2 + l2) + m2n = 0
Hence, the line lx + my + n = 0 will be tangent to the parabola y 2 = 4ax if ln = am2 and
will be normal if al (2m2 + l2) + m2n = 0.
EXERCISE 3. 4 (B)
1. Find the equations to the tangent and the normal of the parabola in each of the following
cases:
(a) y2 = 16x at (1, 4)
(b) y2 = 5x, the tangent is parallel to the line y = 4x + 1. Also, find the point of contact.
(c) y2 = 8x, the tangent is perpendicular to the line y = x + 2
(d) y2 = 4ax at the extremities of the latusrectum.
2. Show that the pair of tangents to the parabola y2 = 24x from (–6, 9) are at right angles.

(Approved by CDC Nepal) Analytical Geometry | 299


3. A tangent to the parabola y 2 = 4x makes an angle of 45° with the line 2x + y = 0, find its
equation and the point of contact.
4. Prove that the lines y = x + 3 and x + y + 3 = 0 are the equations of tangents at the
extremities of the latus rectum of a parabola y2 = 12x.
5. Find the value of k so that 3y = 9x + k will be tangent to the parabola y2 = 8x.
6. Find the value of a so that the line x + y + 2 = 0 will be tangent to the parabola y 2 =
4ax.
ANSWERS
1. (a) y = 2x + 2, x + 2y – 9 = 0
5 5 5
(b) y = 4x + 16 , 4x – y = 90, 64  8
 
(c) x + y + 2 = 0 and y = x – 6
(d) y = x + a, x+y–3a=0 at (a, 2a) and x + y + a = 0 and x – y – 3a = 0 at (a, – 2a)
1 x 1 2
3. y = 3x + 3 , y = –3 –3 and 9  3 , (9, –6) 5. 2 6. 2
 

Objectives Questions
Circle the correct answers
1. The parametric equations x = at2 and y = 4at, t  R represents
a. parabola b. hyperbola c. ellipse d. circle
2
2. The tangent to the parabola y = 4ax at point (a,x) makes with x-axis an angle which is equal to
   
a. b. c. d.
6 4 3 3
3. The length of latus rectum of the parabola y2 = 4ax if the line 2x+3y=1 touches it is
8 4
a. b. 8 c. d. 4
9 3
2
4. If x + y = k is a normal to the parabola y = 12x then the value of k is
a. 3 b. 6 c. 9/2 d. 9
5. The locus of the point of interesection of the perpendicular tangents to y2 = 4ax is
a. x + a = 0 b. x - a = 0 c. y + a = 0 d. y - a = 0
6. The equation tangent to the parabola 4y2+6x=8y+7 at its vertex is
a. 6x = 1 b. 6x + 11 = 0 c. y = 1 d. y + 1 = 0
2
7. The slopes of normal to the parabola y = 8x passing through (18,12) are
a. -1, -2, 3 b. 1, 2, -3 c. 1, 2, 3 d. 0, 1, -1
8. If x+y-a = 0 is a tangent to the parabola y2 - y + x = 0 then its point of contact is
a. (a,0) b. (1,0) c. (0,1) d. (0,-1)
9. If (2,-3) is the one end of a foal chord of a parabola y2 = 3x, then the other end of the chord is
a. (2,4) b. (4,8) c. (2,8) d. (32,32)
2
10. Which one of the following couves cuts the parabola y =4ax at rt angles?
a. x2+y2 = a2 b. y = e-x/2a c. y2 = ax d. x2 = 4ay
Answers:
1 a 2 b 3 a 4 d 5 a
6 a 7 b 8 c 9 d 10 b



300 | Pioneer Mathematics Grade XI (Approved by CDC Nepal)

You might also like